2022/2023 HESI Med-Surg RN Custom Exam (for Med Surg II Class) Pics & Q&As Included (A+) (Verified Answers)

When describing patient education approaches, the nurse educator would explain that informal teaching is an approach that

a. follows formalized plans
b. has standardized content
c. often occurs one-to-one
d. addresses group needs
C. Informal teaching is individualized one on one teaching which represents the majority of patient education done by nurses that occurs when an intervention is explained or a question is answered. Group needs are often the focus of formal patient education courses or classes. Informal teaching does not necessarily follow a specific formalized plan. It may be planned with specific content, but it is individualized responses to patient needs. Formal teaching involves the use of a curriculum/course plan with standardized content.

A patient expresses a strong interest in returning to their work, family, and hobbies after having a stroke. Which theory type would the nurse use to develop a plan of care for the best results of this patient’s motivation style?

a. field
b. biological
c. cognitive
d. sociologic
C. Cognitive theorists believe that attention, relevance, confidence, and satisfaction (ARCS) are the conditions that, when integrated, motivate someone to learn. Field theorists place significance on how achievement, power, the need for affiliation, and avoidance motives influence individual behavior. Sociologic theories are not involved in motivation.

The nurse is assessing a group of clients. Which clients are at greater risk for hypothermia or frostbite? (select all that apply)

a. an older woman with hypertension
b. a young man with a body mass index of 42
c. a young many who has just consumed six martinis
d. an older man who smokes a pack of cigarettes a day
e. a young woman who is anorexic
f. a young woman who is diabetic
C, D, E, F

clients with poor nutrition, fatigue, and multiple chronic illnesses are at greater risk for hypothermia. Clients who smoke, consume alcohol, or have impaired peripheral circulation have a higher incidence of frostbite.

Which statement made by a nurse represents the need for further education regarding pain management in older adult clients?

a. older adults tend to report pain less often than younger adults
b. older clients usually have more experience with pain than younger clients
c. older adults are at greatest risk for under treated pain
d. older clients have a different pain mechanism and do not feel it as much
D

There is no evidence to support the idea that older adult clients perceive pain any differently than younger clients. The other statements are accurate regarding older clients and pain.

The nurse is working at a first aid booth for a spring training game on a hot day. A spectator comes in, reporting that he is not feeling well. Vital signs are temp 104.1 F, pulse 132 BPM, respirs 26 breaths/min, and blood pressure 106/66 mm Hg. He trips over his feet as the nurse leads him to a cot. What is the priory action of the nurse?

a. admin tylenol 650 mg orally
b. encourage rest, and reassess in 15 minutes
c. sponge the victim with cool water and remove his shirt
d. encourage drinking of cool water or sports drink
C

The spectator shows signs of heat stroke, which is a medical emergency. The spectator should be transported to the ED ASAP. The nurs should take actions to lower his body temp in teh meantime by removing his shirt and sponging his body with cool water. Lowering body temp by drinking cool fluids or taking acetaminophen is not as effective in an emergency situation. The client needs to be cooled quickly and is a priority for treatment

The client is receiving an IV of 60 mEq of potassium chloride ina 1000 mL solution of dextrose 5% in 0.45% saline. The client states that the area around the IV site burns. What intervention does the nurse perform first?

a. assess for a blood return
b. notify the physician
c. document the finding
d. stop the IV infusion
D

Potassium is a severe tissue irritant. The safest action is to discontinue the solution that contains the potassium and discontinue the IV altogether, in which case the client would need another site started. Assessing for a blood return may or may not be successful. The solution could be diluted (less potassium) and the rate could be slowed once it is determined that the needle is in the vein.

A nurse is caring for an older adult client who lives alone. Which economic situation presents the most serious problem for this client?

a. costs of creating a living will
b. stock market fluctuations
c. increased provider benefits
d. social security as the basis of income
D

Older adults on fixed incomes are unable to adjust their income to meet rising costs associated with meeting basic needs

Controlling pain is important to promoting wellness. Unrelieved pain has been associated with

a. prolonged stress response and a cascade of harmful effects system wide.
b. decreased tumor growth and longevity
c. large tidal volumes and decreased lung capacity
d. decreased carbohydrate, protein, and fat destruction
A

Pain triggers a number of physiologic stress responses in the human body. Unrelieved pain can prolong the stress response and produce a cascade of harmful effects in all body systems. The stress response causes the endocrine system to release excessive amounts of hormones, such as cortisol, catecholamines, and glucagon. Insulin and testosterone levels decrease. Increased endocrine activity in turn initiates a number of metabolic processes, in particular, accelerated carbohydrate, protein, and fat destruction, whcih can result in weight loss, tachycardia, increased respiratory rate, shock, and even death. The immune system is also affected by pain as demonstrated by research showing a link between unrelieved pain and a higher incidence of nosocomial infections and increased tumor growth. Large tidal volumes are not associated with pain while decreased lung capacity is associated with unrelieved pain. Decreased tumor growth and longevity are not associated with unrelieved pain. Decreased carbs, protein, and fat are not associated with pain or stress response.

Which intervention in a client with dehydration induced confusion is most likely to relieve the confusion?
a. increasing the IV flow rate to 250 mL/hr
b. applying oxygen by mask or nasal cannula
c. placing the client in a high Fowler’s position
d. Measuring intake and output every four hours
A
Dehydration most frequently leads to poor cerebra perfusion and cerebral hypoxia, causing confusion. Applying oxygen can reduce confusion, even if perfusion is still less than optimum. Increasing the IV flow rate would increase perfusion. However, depending on the degree of dehydration, rehydrating the person too rapidly with IV fluids can lead to cerebral edema.

Which client is at greatest risk for dehydration?

a. younger adult client on bedrest
b. older adult client receiving hypotonic IV fluid
c. older adult client with cognitive impairment
d. younger adult client receiving hypertonic IV fluid
C

Older adults, because they have less total body water than younger adults, are at greater risk for development of dehydration. Anyone who is cognitively impaired and cannot obtain fluids independently or cannot make his or her need for fluids known is at high risk for dehydration

A nurse is caring for several clients. Which client does the nurse assess most carefully for hyperkalemia?

a. client with type 2 diabetes taking an oral anti-diabetic agent
b. client with heart failure using a salt substitute
c. client taking a thiazide diuretic for hypertension
d. client taking non-steroidal anti-inflammatory drugs daily
B

Many salt substitutes are composed of potassium chloride. Heavy use cna contribute to the development of hyperkalemia. The client should be taught to read labels and to choose a salt substitute that does not contain potassium. NSAIDs promote the retention of sodium but not potassium.

An older adult client presents with signs and symptoms related to dig toxicity. Which age related change may have contributed to this problem?

a. decreased renal blood flow
b. increased gastrointestinal motility
c. decreased ratio of adipose tissue to lean body mass
d. increased total body water
A

Decreased renal blood flow and reduced glomerular filtration can result in slower medication excretion time, potentially leading to toxic drug accumulation. Aging results in decreased total body water and gastrointestinal motility and an increase in the ratio of adipose tissue to lean body mass, but is not related to dig toxicity.

A client is being treated for dehydration. Which statement made by the client indicates understanding of this condition?

a. I will use a salt substitute when making and eating my meals.
b. I must drink a quart of water or other liquid each day.
c. I will not drink liquids after 6 PM so I won’t have to get up at night.
d. I will weigh myself each morning before I eat or drink.
D

Because 1 L of water weighs 1 kg, change in body weight is a good measure of excess fluid loss or fluid retention. Weight loss greater than 0.5 lb daily is indicative of excessive fluid loss. The other statements are not indicative of practices that will prevent dehydration.

The nurse notes that the handgrip of the client with hypokalemia has diminished since the previous assessment one hour ago. Which intervention by the nurse is the priority?

a. assess the client’s respiratory rate, rhythm, and depth
b. document findings and monitor the client
c. measure the client’s pulse and blood pressure
d. call the health care provider
A

In a client with hypokkalemia, progressive skeletal muscle weakness is associated with increasing severity of hypokalemia. The most life-threatening complication of hypokalemia is respiratory insufficiency. It is imperative for the nurse to perform a respiratory assessment first to make sure that the client is not in immediate jeopardy. Next, the nurse would call the health care provider to obtain orders for potassium replacement.

The physician orders Lasix (furosemide) 60 mg po every day for your patient. On hand you have Lasix 40 mg. How many tablets will you give the patient?

a. 3
b. 1
c. 1 1/2
d. 2 1/5
C

60/40 (desired/have)

A client has been taught to restrict dietary sodium. Which food selection by the client indicates to the nurse that teaching has been effective?

a. a grilled cheese sandwich with tomato soup
b. Chinese take-out, including steamed rice
c. a chicken leg, one slice of bread with butter, and steamed carrots
d. slices of ham and cheese on whole grain crackers
C

Clients on restricted sodium diets generally should avoid processed, smoked, and pickled foods and those with sauces and other condiments. Foods lowest in sodium include fish, poultry, and fresh produce. The chinese food likely would have soy sauce, the tomato soup is processed, and the crackers are a snack food – a category of foods often high in sodium.

When a client is assessed, which behavior best indicates that he or she is experiencing changes associated with acute pain?

a. inability to concentrate
b. expressed hopelessness
c. psychosocial withdrawal
d. anger and hostility
A

The characteristics most common to chronic pain are psychosocial withdrawal, anger and hostility, depression, and hopelessness. The inability to concentrate is associated much more with acute pain, before any physiologic or behavioral adaptation has occurred.

A nurse is caring for several clients at risk for overhydration. The nurse assesses the older client with which finding first?

A) Has had diabetes mellitus for 12 years
B) Had abdominal surgery and has a nasogastric tube
C) Just received 3 units of packed red blood cells
D) Uses sodium-containing antacids frequently
C

Blood replacement therapy involves intravenous fluid administration, which inherently increases the risk for overhydration. The fact that the fluid consists of packed red blood cells greatly increases the risk, because this fluid increases the colloidal oncotic pressure of the blood, causing fluid to move from interstitial and intracellular spaces into the plasma volume. An older adult may not have sufficient cardiac or renal reserve to manage this extra fluid.

The client with a stroke was admitted to a medical-surgical unit. Which tasks does the nurse delegate to the unlicensed assistive personnel?

A) Assess level of consciousness.
B) Evaluate the pulse oximetry reading.
C) Assist the client with meals.
D) Complete the nursing care plan.
C

The nurse needs to know the five rights of delegation: right task, right circumstances, right person, right communication, and right supervision. Unlicensed assistive personnel can help with feeding, but only the nurse can care plan, assess the level of consciousness, and evaluate the oxygenation of the client.

Interrelated concepts to the professional nursing role a nurse manager would consider when addressing concerns about the quality of patient education include:

A) adherence.
B) developmental level.
C) motivation.
D) technology.
D

The interrelated concepts to the professional role of a nurse include health promotion, leadership, technology/informatics, quality, collaboration, and communication. Adherence, culture, developmental level, family dynamics, and motivation are considered interrelated concepts to patient attributes and preference.

During orientation to an emergency department, the nurse educator would be concerned if the new nurse listed which of the following as a risk factor for impaired thermoregulation?
A) Temperature extremes
B) Occupational exposure
C) Impaired cognition
D) Physical agility
D

Physical agility is not a risk factor for impaired thermoregulation. The nurse educator would use this information to plan additional teaching to include medical conditions and gait disturbance as risk factors for hypothermia, because their bodies have a reduced ability to generate heat. Impaired cognition is a risk factor. Recreational or occupational exposure is a risk factor. Temperature extremes are risk factors for impaired thermoregulation.

An older adult client is in physical restraints. Which intervention by the nurse is the priority?

A) Assess the client hourly while keeping the restraints in place.
B) Assess the client once each shift, releasing the restraints for feeding.
C) Assess the client twice each shift while keeping the restraints in place.
D) Assess the client every 30 to 60 minutes, releasing restraints every 2 hours.
D

The application of restraints can have serious consequences. Thus, the nurse should check the client every 30 to 60 minutes, releasing the restraints every 2 hours for positioning and toileting. The other answers would not be appropriate because the client would not be assessed frequently enough, and circulation to the limbs could be compromised. Assessing every hour and releasing the restraints every 2 hours is in compliance with federal policy for monitoring clients in restraints.

The nurse is assessing a client with a long-term history of arthritic pain. Assessment reveals a heart rate of 115 beats/min and blood pressure of 170/80 mm Hg. Which intervention will the nurse carry out first?

A) Administer blood pressure medication.
B) Administer a drug to lower the heart rate.
C) Continue to assess for possible causes of elevated vital signs.
D) Assess whether the client needs anti-arthritis medication.
C

Arthritis is categorized as chronic pain. With chronic pain, the body adapts by blocking the sympathetic nervous system; this normally causes tachycardia and increased blood pressure. Therefore, this client’s high blood pressure and heart rate are not caused by chronic pain and may be a result of a more acute type of pain. Therefore, the best intervention is for the nurse to establish whether the client is having pain other than arthritic pain, and then to decide which intervention should be carried out.

The nurse is assigned to care for the following four clients who have the potential for having pain. Which client is most likely not to be treated adequately for this problem?

A) Middle-aged woman with a fractured arm
B) Client with expressive aphasia
C) Younger adult with metastatic cancer
D) Client who has undergone an appendectomy
B

Populations at highest risk for inadequate pain treatment include older adults, minorities, and those with a history of substance abuse. Nonverbal clients are very difficult to assess for pain because self-report is not possible, and the nurse needs to rely on client behaviors or surrogate reporting.

Before surgery, the nurse observes the client listening to music on the radio. Based on this observation, the nurse may try which nonpharmacologic intervention for pain relief in the postoperative setting?

A) Cutaneous skin stimulation
B) Imagery
C) Radiofrequency ablation
D) Hypnosis
B

Imagery is a form of distraction in which the client is encouraged to visualize about some pleasant or desirable feeling, sensation, or event. Behaviors that are helpful in assessing a client’s capacity for imagery include being able to listen to music or other auditory stimuli.

What interrelated constructs facilitate a nurse to become culturally competent?

A) Cultural desire, self-awareness, cultural knowledge, and cultural skill
B) Cultural desire, self-awareness, cultural knowledge, and cultural diversity
C) Cultural desire, self-awareness, cultural knowledge, and cultural identity
D) Cultural diversity, self-awareness, cultural skill, and cultural knowledge
A

The process of cultural competence consists of four interrelated constructs: cultural desire, self-awareness, cultural knowledge, and cultural skill. Cultural diversity in the context of health care refers to achieving the highest level of health care for all people by addressing societal inequalities and historical and contemporary injustices. Cultural identity is the norms, values, beliefs, and behaviors of a culture learned through families and group members.

The emphasis on understanding cultural influence on health care is important because of:

A) disability entitlements.
B) HIPAA requirements.
C) litigious society.
D) increasing global diversity.
D

Culture is an essential aspect of health care because of increasing diversity. Disability entitlements refer to defined benefits for eligible mental or physically disabled beneficiaries in relation to housing, employment, and health care. HIPAA requirements refers to the HIPAA Privacy Rule, which protects the privacy of individually identifiable health information; the HIPAA Security Rule, which sets national standards for the security of electronic protected health information; and the confidentiality provisions of the Patient Safety Rule, which protect identifiable information being used to analyze patient safety events and improve patient safety.
Litigious society refers to excessively ready to go to law or initiate a lawsuit.

The patient’s laboratory report today indicates severe hypokalemia, and the nurse has notified the physician. Nursing assessment indicates that heart rhythm is regular. What is the most important nursing intervention for this patient now?

A) Examine sacral area and patient’s heels for skin breakdown due to potential edema.
B) Establish seizure precautions due to potential muscle twitching, cramps, and seizures.
C) Institute fall precautions due to potential postural hypotension and weak leg muscles.
D) Raise bed side rails due to potential decreased level of consciousness and confusion.
C

Hypokalemia can cause postural hypotension and bilateral muscle weakness, especially in the lower extremities. Both of these increase the risk of falls. Hypokalemia does not cause edema, decreased level of consciousness, or seizures.

A nurse is assessing clients for fluid and electrolyte imbalances. Which client is at greatest risk for developing hyponatremia?

A) Client taking digoxin (Lanoxin)
B) Client who is NPO receiving intravenous D5W
C) Client taking ibuprofen (Motrin)
D) Client taking a sulfonamide antibiotic
B

D5W contains no electrolytes. Because the client is not taking any food or fluids by mouth, normal sodium excretion can lead to hyponatremia. The antibiotic, Motrin, and digoxin will not put a client at risk for hyponatremia.

The nurse accidentally administers 10 mg of morphine intravenously to a client who had been given another dose of morphine, 5 mg IV, about 30 minutes earlier. What action must the nurse be prepared to take?

A) Assist with intubation.
B) Monitor pain level.
C) Administer oxygen.
D) Administer naloxone (Narcan).
D

A combined dose of 15 mg of morphine may cause severe respiratory depression in some clients. Naloxone is an opioid antagonist that can be used (intravenously) as the first intervention to reverse respiratory depression due to a morphine overdose. Then administration of oxygen may be needed if the client’s oxygen saturation decreases. Intubation may occur if the client does not respond to the Narcan, and respiratory depression becomes a respiratory arrest. Naloxone may be repeated, but the pain level of the client needs to be monitored because Narcan can promote withdrawal symptoms.

Which action does the nurse teach a client to reduce the risk for dehydration?

A) Avoiding the use of glycerin suppositories to manage constipation
B) Maintaining a daily oral intake approximately equal to daily fluid loss
C) Restricting sodium intake to no greater than 4 g/day
D) Maintaining an oral intake of at least 1500 mL/day
B

Although a fixed oral intake of 1500 mL daily is good, the key to prevention of dehydration is to match all fluid losses with the same volume for fluid intake. This is especially true in warm or dry environments, or when conditions result in greater than usual fluid loss through perspiration or ventilation.

A client is taking furosemide (Lasix) and becomes confused. Which potassium level does the nurse correlate with this condition?

A) 2.9 mEq/L
B) 5.0 mEq/L
C) 6.0 mEq/L
D) 3.8 mEq/L
A

Hypokalemia decreases cerebral function and is manifested by lethargy, confusion, inability to perform problem-solving tasks, disorientation, and coma. Normal potassium levels are 3.5 to 5.0 mEq/L. At 2.9 mEq/L, potassium is too low, and this could lead to neurologic manifestations.

The most appropriate measure for a nurse to use in assessing core body temperature when there are suspected problems with thermoregulation is a(n)
A) rectal thermometer.
B) tympanic membrane sensor.
C) temporal thermometer scan.
D) oral thermometer.
A

The most reliable means available for assessing core temperature is a rectal temperature, which is considered the standard of practice. An oral temperature is a common measure but not the most reliable. A temporal thermometer scan has some limitations and is not the standard. The tympanic membrane sensor could be used as a second source for temperature assessment.

A client presents to the emergency department after prolonged exposure to the cold. The client is shivering, has slurred speech, and is slow to respond to questions. Which intervention will the nurse prepare for this client FIRST?

A) Continuous arteriovenous rewarming
B) Dry clothing and warm blankets
C) Peritoneal lavage with warmed normal saline
D) Administration of warmed IV fluids
B

Mild hypothermia is manifested by shivering, slurred speech, poor muscular coordination, and impaired cognitive abilities. Mild hypothermia may be treated with dry clothing and warm blankets. Rewarming should occur slowly by removing wet clothing and providing dry warm blankets first. Other treatments are secondary and should be used to treat moderate to severe hypothermia.

The Joint Commission focuses on safety in health care. Which action by the nurse reflects The Joint Commission’s main objective?

A) Performing range-of-motion exercises on the client three times each day
B) Assessing the client’s respirations when administering opioids
C) Delegating to the nursing assistant to give the client a complete bath daily
D) Ensuring that the client is eating 100% of the meals served to him or her
B

It is important for the nurse to assess respirations of the client when administering opioids because of the possibility of respiratory depression. The other interventions may or may not be necessary in the care of the client and do not focus on safety.

What is a priority nursing intervention to prevent falls for an older adult client with multiple chronic diseases?

A) Requesting that a family member remain with the client to assist in ambulation
B) Keeping all four siderails up while the client is in bed
C) Placing the client in restraints to prevent movement without assistance
D) Providing assistance to the client in getting out of the bed or chair
D

Advanced age and multiple illnesses, particularly those that result in alterations in sensation, such as diabetes, predispose this client to falls. The nurse should provide assistance to the client with transfer and ambulation to prevent falls. The client should not be restrained or maintained on bedrest without adequate indication. Although family members are encouraged to visit, their presence around the clock is not necessary at this point.

The nurse is caring for four clients. Which client assessment is the most indicative of having pain?

A) Client stating that he is “anxious”
B) Heart rate of 105 beats/min and restlessness
C) Blood pressure 150/70 mm Hg and sleeping
D) Postoperative client with a neck incision
B

At times clients are unable to verbalize that they are in pain but there are indicators that the client may have acute pain such as increased heart rate, increased blood pressure, increased respirations, sweating, restlessness, and overall distress. All the other distractors could indicate clients who have the potential for being in pain, but restlessness with tachycardia is the most indicative.

The Institute for Healthcare Improvement (IHI) identified interventions to save client lives. Which actions are within the scope of nursing practice to improve quality of care?

A) Prescribe aspirin for a client who presents with an acute myocardial infarction
B) Insert a central line to give intravenous fluid to a dehydrated client.
C) Use sterile technique when changing dressings on a new surgical site.
D) Intubate a client whose oxygen saturation is 92%.
C

The only intervention identified within the scope of nursing practice is to use sterile technique. Central line insertion, intubation, and prescription are functions of the physician.

Which is most indicative of pain in an older client who is confused? (Select all that apply).

A) Screaming
B) Decreased blood pressure
C) Crying
D) Decreased respirations
E) Facial grimace
F) Restlessness
A,C,E,F

No one scale has been found to be the best tool to use in pain assessment for adults with cognitive impairment. Facial expression, motor behavior, mood, socialization, and vocalization are common indicators of pain in cognitively impaired adults. In acute pain, nonverbal indicators of pain could include increased blood pressure and respirations.

The nursery nurse identifies a newborn at significant risk for hypothermic alteration in thermoregulation because the patient is:

A) large for gestational age.
B) well nourished.
C) born at term.
D) low birth weight.
D

Low birth weight and poorly nourished infants (particularly premature infants) and children are at greatest risk for hypothermia. A large for gestational age infant would not be malnourished. An infant born at term is not considered at significant risk. A well nourished infant is not at significant risk.

The nurse is assessing a patient’s functional ability. Which activities most closely match the definition of functional ability?

A) Healthy individual, college educated, travels frequently, can balance a checkbook
B) Healthy individual, works out, reads well, cooks and cleans house
C) Healthy individual, volunteers at church, works part time, takes care of family and house
D) Healthy individual, works outside the home, uses a cane, well groomed
C

Functional ability refers to the individual’s ability to perform the normal daily activities required to meet basic needs; fulfill usual roles in the family, workplace, and community; and maintain health and well-being. The other options are good; however, each option has advanced or independent activities in the context of the option.

Which action demonstrates that the nurse understands the purpose of the Rapid Response Team?

A) Documenting all changes observed in the client and maintaining a postoperative flow sheet
B) Monitoring the client for changes in postoperative status such as wound infection
C) Notifying the physician of the client’s change in blood pressure from 140 to 88 mm Hg systolic
D) Notifying the physician of the client’s increase in restlessness after medication change
C

The Rapid Response Team (RRT) saves lives and decreases the risk for harm by providing care to clients before a respiratory or cardiac arrest occurs. Although the RRT does not replace the Code Team, which responds to client arrests, it intervenes rapidly for those who are beginning to decline clinically. It would be appropriate for the RRT to intervene when the client has experienced a 52-point drop in blood pressure. Monitoring the client’s postoperative status, maintaining a postoperative flow sheet, and notifying the physician of a change in the client’s status after a medication change would not be considered activities of the Rapid Response Team.

An older client just returned from surgery and is rating pain as “8” on a 0 to 10 scale. Which medications are unsafe choices for treatment of severe pain in this older adult? (Select all that apply.)

A) Morphine (Durmorph)
B) Meperidine (Demerol)
C) Propoxyphene (Darvocet)
D) Methadone (Dolophine)
E) Codeine
B,C,D,E

Meperidine, propoxyphene, and codeine are not recommended for older clients because toxic metabolites may accumulate. Codeine may cause constipation as well. Methadone has an extremely long half-life (24 to 36 hours) and has a high potential for sedation and respiratory depression. Morphine is considered the gold standard and may be used in the older adult while monitoring for sedation and respiratory depression is conducted.

An emergency department (ED) nurse gives report on a client who is being transferred to the medical-surgical floor. Because of an identified risk for suicide, the ED nurse suggests that the floor nurse contact a sitter and behavioral health. This statement represents which part of the SBAR hand-off?

A) Situation
B) Recommendation
C) Background
D) Assessment
B

The ED nurse is giving recommendations to the medical-surgical floor nurse about interventions to start for the client who is being transferred. No communication is provided in the SBAR report about the situation, background, or assessment.

Understanding classifications of pain helps nurses develop a plan of care. A 62-year-old male has fallen while trimming tree branches sustaining tissue injury. He describes his condition as an aching, throbbing back. This is characteristic of:

A) mixed pain syndrome.
B) chronic pain.
C) neuropathic pain.
D) nociceptive pain.
D

Nociceptive pain refers to the normal functioning of physiological systems that leads to the perception of noxious stimuli (tissue injury) as being painful. Patients describe this type of pain as aching, cramping, or throbbing. Neuropathic pain is pathologic and results from abnormal processing of sensory input by the nervous system as a result of damage to the brain, spinal cord, or peripheral nerves. Patients describe this type of pain as burning, sharp, and shooting. Chronic pain is constant and unrelenting such as pain associated with cancer. Mixed pain syndrome is not easily recognized, is unique with multiple underlying and poorly understood mechanisms like fibromyalgia and low back pain.

The new nurse is caring for a client with a high temperature. Which action should the nurse perform FIRST?

A) Obtaining a fan from central supply for the client’s room
B) Monitoring the client’s temperature more often than ordered
C) Sponging the client while monitoring for shivering
D) Apply cool packs to the client’s axillae and groin
D

The use of fans is discouraged to promote cooling in a febrile client because the fan can disperse pathogens. The other actions are appropriate.

A patient has been newly diagnosed with hypertension. The nurse assesses the need to develop a collaborative plan of care that includes a goal of adhering to the prescribed regimen. When the nurse is planning teaching for the patient, which is the most important initial learning goal?

A) The patient will demonstrate coping skills needed to manage hypertension.
B) The patient will verbalize the side effects of treatment.
C) The patient will select the type of learning materials they prefer.
D) The patient will verbalize an understanding of the importance of following the regimen.
C

Adults learn best when given information they can understand that is tailored to their learning styles and needs. Verbalizing an understanding is important; however, the nurse will first need to teach the patient.

When reviewing the purposes of a family assessment, the nurse educator would identify a need for further teaching if the student responded that family assessment is used to gain an understanding of the family.

A) development.
B) function.
C) structure.
D) political views.
D

An understanding of the political views of family members is not a primary purpose of a family assessment. A family assessment provides the nurse with information and an understanding of family dynamics. This is important to nurses for the provision of quality health care. A family assessment provides an understanding of family development, function, and structure.

The client was given 15 mg of morphine IM for postsurgical pain. When the nurse checks the client for pain relief 1 hour later, the client is sleeping and has a respiratory rate of 10 breaths/min. What is the nurse’s first action?

A) Administering oxygen by nasal cannula
B) Documenting the findings and continuing to monitor
C) Arousing the client by calling his or her name
D) Administering naloxone (Narcan) IV push
C

Many clients experience some degree of respiratory depression with opioid analgesics. If the client can be aroused with minimally intrusive techniques and the rate of respiration is increased spontaneously, no further intervention is required.

The physician orders Lanoxin(digoxin)0.375 mg po every day. On hand you have 0.25mg/5 mL. How many mL would you give your patient?

A) 8 mL
B) 7.5 mL
C) 7 mL
D) 5.5 mL
B

The nurse is admitting an older adult with decompensated congestive heart failure. The nursing assessment reveals adventitious lung sounds, dyspnea, and orthopnea. The nurse should question which doctor’s order?

A) KCl 20 mEq PO two times per day
B) Intravenous (IV) 500 mL of 0.9% NaCl at 125 mL/hr
C) Oxygen via face mask at 8 L/min
D) Furosemide (Lasix) 20 mg PO now
B

A patient with decompensated heart failure has extracellular fluid volume (ECV) excess. The IV of 0.9% NaCl is normal saline, which should be questioned because it would expand ECV and place an additional load on the failing heart. Diuretics such as furosemide are appropriate to decrease the ECV during heart failure. Increasing the potassium intake with KCl is appropriate, because furosemide increases potassium excretion. Oxygen administration is appropriate in this situation of near pulmonary edema from ECV excess.

The priority nursing intervention for a patient suspected to be hypothermic would be to:

A) hydrate with intravenous (IV) fluids.
B) remove wet clothes.
C) assess vital signs.
D) provide a warm blanket.
B

The first thing to do with a patient suspected to be hypothermic is to remove wet clothes, because heat loss is five times greater when clothing is wet. Assessing vital signs is important, but the wet clothes should be removed first. Hydration is very important with hyperthermia and the associated danger of dehydration, but there is not a similar risk with hypothermia. A warm blanket over wet clothes would not be an effective warming strategy.

The nurse admitting a patient to the emergency department on a very hot summer day would suspect hyperthermia when the patient demonstrates:

A) slow capillary refill.
B) red, sweaty skin.
C) low pulse rate.
D) decreased respirations.
B

With hyperthermia, vasodilatation occurs causing the skin to appear flushed and warm or hot to touch. There is an increased respiration rate with hyperthermia. The heart rate increases with hyperthermia. With hypothermia there is slow capillary refill.

Why does the nurse always ask the client his or her pain level after taking routine vital signs?

A) To follow McCaffery’s guidelines on pain management
B) To ensure that pain assessment occurs on a regular basis
C) To determine the need for more frequent vital sign measurement
D) To determine whether pain is influencing blood pressure and heart rate
B

Making pain the fifth vital sign allows more frequent and accurate assessment, which can contribute to better pain management.

The nurse observes skin tenting on the back of the older adult client’s hand. Which action by the nurse is most appropriate?

A) Examine dependent body areas.
B) Notify the physician.
C) Document the finding and continue to monitor.
D) Assess turgor on the client’s forehead.
D

Skin turgor cannot be accurately assessed on an older adult client’s hands because of age-related loss of tissue elasticity in this area. Areas that more accurately show skin turgor status on an older client include the skin of the forehead, chest, and abdomen. These should also be assessed, rather than merely examining dependent body areas. Further assessment is needed rather than only documenting, monitoring, and notifying the physician.

The nurse is assessing a client who has undergone a transurethral resection of the prostate (TURP). Which assessment finding requires immediate action by the nurse?

A) Having the urge to void continuously while the catheter is inserted
B) Passing small blood clots after catheter removal
C) Having bright red drainage with multiple blood clots
D) Experiencing urinary frequency after catheter removal
C

A client who undergoes a TURP is at risk for bleeding during the first 24 hours after surgery. Passage of small blood clots and tissue debris, urinary frequency and leakage, and the urge to void continuously while the client still has the catheter inserted are all considered to be expected complications of the procedure. They will resolve as the client continues to recover and the catheter is removed. However, the presence of bright red blood with clots indicates arterial bleeding and should be reported to the provider.

Which finding puts a client at greatest risk for wound infection?

A) Presence of a deep wound
B) Coexisting medical conditions
C) Immune compromised status
D) Severely reddened skin
C

A compromised immune system puts a client at greatest risk for infection. Although all the other options might increase the client’s susceptibility, the one with the greatest potential impact is being immune compromised.

The nurse is assessing a client with an early onset of multiple sclerosis (MS). Which clinical manifestations does the nurse expect to see?

A) Nystagmus & Diplopia
B) Hyperresponsive reflexes
C) Excessive somnolence
D) Heat intolerance
A

Early signs and symptoms of MS include changes in motor skills, vision, and sensation. The other manifestations are later signs of MS.

The nurse determines that a client has a Braden Scale score of 9. Which is the nurse’s best intervention related to this assessment?

A) Increase the client’s fluid intake.
B) Consult with the health care provider.
C) Reassess the client in 3 days.
D) Document the finding per protocol.
B

A score of 11 or less on the Braden Scale indicates severe risk for pressure ulcer development in terms of decreased sensory perception, exposure to moisture, decreased independent activity, decreased mobility, poor nutrition, and chronic exposure to friction and shear. The nurse needs to consult with the health care provider to relay this information and to obtain more aggressive skin protection measures than are currently provided.

While planning care for a patient experiencing fatigue due to chemotherapy, which of the following is the most appropriate nursing intervention?

A) Completing all nursing care in the evening when the patient is more rested
B) Completing all nursing care in the morning so the patient can rest the remainder of the day
C) Limiting visitors, thus promoting the maximal amount of hours for sleep
D) Prioritization and administration of nursing care throughout the day
D

Pacing activities throughout the day conserves energy, and nursing care should be paced as well. Fatigue is a common side effect of cancer and treatment; and while adequate sleep is important, an increase in the number of hours slept will not resolve the fatigue. Restriction of visitors does not promote healthy coping and can result in feelings of isolation.

A diabetic client has numbness and reduced sensation. Which intervention does the nurse teach this client to prevent injury?

A) “Use a bath thermometer to test the water temperature.”
B) “Examine your feet daily using a mirror.”
C) “Wear white socks instead of colored socks.”
D) “Rotate your insulin injection sites.”
A

Clients with diminished sensory perception can easily experience a burn injury when bath water is too hot. Instead of checking the temperature of the water by feeling it, they should use a thermometer. Examining the feet daily does not prevent injury, although daily foot examinations are important to find problems so they can be addressed. Rotating insulin and wearing white socks also will not prevent injury.

Which client does the nurse assess to be at greatest risk for pressure ulcer development?

A) Client who requires assistance with ambulation
B) Incontinent client with limited mobility
C) Client with hypertension on multiple medications
D) Client who has pneumonia
B

Being immobile and being incontinent are two significant risk factors for the development of pressure ulcers. Clients with pneumonia and hypertension do not have specific risk factors. The client who needs assistance with ambulation might be at moderate risk if he or she does not move about much, but having two risk factors makes the last option the person at highest risk.

The nurse is instructing the nursing assistant to prevent pressure ulcers in a frail older patient; the nursing assistant understands the instruction when she agrees to:

A) bathe and dry the skin vigorously to stimulate circulation.
B) limit intake of fluid and offer frequent snacks.
C) turn the patient at least every 2 hours.
D) keep the head of the bed elevated 30 degrees.
C

The patient should be turned at least every 2 hours as permanent damage can occur in 2 hours or less. If skin assessment reveals a stage I ulcer while on a 2-hour turning schedule, the patient must be turned more frequently. Limiting fluids will prevent healing; however, offering snacks is indicated to increase healing particularly if they are protein based, because protein plays a role in healing. Use of doughnuts, elevated heads of beds, and overstimulation of skin may all stimulate, if not actually encourage, dermal decline.

The client with type 2 diabetes has recently been changed from the oral antidiabetic agents glyburide (Micronase) and metformin (Glucophage) to glyburide-metformin (Glucovance). The nurse includes which information in the teaching about this medication?

A) “Glucovance is more effective than glyburide and metformin.”
B) “Your diabetes is improving and you now need only one drug.”
C) “Glucovance contains a combination of glyburide and metformin.”
D) “Glucovance is a new oral insulin and replaces all other oral antidiabetic agents.”
C

Glucovance is composed of glyburide and metformin. It is given to enhance the convenience of antidiabetic therapy with glyburide and metformin. The other statements are not accurate.

The nurse administers 6 units of regular insulin and 10 units NPH insulin at 7 AM. At what time does the nurse assess the client for problems related to the NPH insulin?

A) 4 PM
B) 11 PM
C) 8 AM
D) 8 PM
A

NPH is an intermediate-acting insulin with an onset of 1.5 hours, peak of 4 to 12 hours, and duration of action of 22 hours. Checking the client at 8:00 AM would be too soon; 8:00 PM and 11:00 PM would be too late.

The nurse is caring for a client who is immobile from a recent stroke. Which intervention does the nurse implement to prevent complications in this client?

A) Teach the client to touch and use both sides of the body.
B) Apply sequential compression stockings.
C) Instruct the client to turn the head from side to side.
D) Position the client with the unaffected side down.
B

To avoid complications of immobility, such as deep vein thrombosis, the nurse applies sequential compression stockings or pneumatic compression boots. Efforts are made to mobilize the client as much as possible, and the client should be repositioned frequently. The other interventions will not prevent complications of immobility.

The nurse is caring for a client who has experienced a stroke. Which nursing intervention for nutrition does the nurse implement to prevent complications from cranial nerve IX impairment?

A) Place the client in high Fowler’s position.
B) Verbalize the placement of food on the client’s plate.
C) Order a clear liquid diet for the client.
D) Turn the client’s plate around halfway through the meal.
A

Cranial nerve IX, the glossopharyngeal nerve, controls the gag reflex. Clients with impairment of this nerve are at great risk for aspiration. The client should be in high Fowler’s position and should drink thickened liquids if swallowing difficulties are present. The client would not have vision problems. Turning the plate around would not prevent a complication, nor would limiting the client’s diet to clear liquids.

Which statement indicates that the client needs more teaching about mucositis?

A) “I will use a soft-bristled toothbrush to prevent trauma.”
B) “I will rinse my mouth with water after every meal.”
C) “I should use an alcohol-based mouth rinse to kill bacteria.”
D) “I cannot use floss because it may irritate my gums.”
C

Mouthwashes that contain alcohol are drying and can exacerbate mucosal irritation, leading to painful mouth sores. Rinsing the mouth with water or normal saline is indicated. Interventions aimed at decreasing risk for trauma or irritation are matters of priority because of inflammation associated with mucositis.

A young woman is being treated with amoxicillin (Amoxil) for a urinary tract infection. Which is the highest priority instruction for the nurse to give this client?

A) “You may experience an irregular heartbeat while on the drug.”
B) “Watch for blood in your urine while taking this drug.”
C) “Use a second form of birth control while on the drug.”
D) “You will experience increased menstrual bleeding while on this drug.”
C

The client should use a second form of birth control because penicillin seems to reduce the effectiveness of estrogen-containing contraceptives. She should not experience increased menstrual bleeding, an irregular heartbeat, or blood in her urine while taking the medication.

The nurse prepares to teach a patient recovering from a myocardial infarction (MI) about combination durg therapy based on “best practice” for controlling hypertension. Which drugs does the nurse include in the teaching plan? SELECT ALL THAT APPLY!!!

A) NSAID’s
B) Aspirin
C) Aldosterone antagonists
D) ACE Inhibitors or ARB’s
E) Central alpha Agonists
F) Beta Blockers
G) Diuretics
B,C,D,F,G

The nurse is caring for a client who is disoriented as the result of a stroke. Which action does the nurse implement to help orient this client?

A) Turn on the television to a 24-hour news station.
B) Provide auditory and visual stimulation simultaneously.
C) Ask the family to bring in pictures familiar to the client.
D) Maintain a calm and quite environment by minimizing visitors.
C

For the client with disorientation, the nurse can request that the family bring in pictures or objects that are familiar to the client. The nurse explains what the object or picture represents in simple terms. These stimuli can be presented several times daily. Visitors can also be familiar stimuli to reorient the client. Too much stimuli and constant stimuli can lead to further confusion.

The nurse is caring for an anorexic client who is severely malnourished. A nasogastric feeding tube is inserted, and tube feedings are started. Which laboratory finding is the best indication that the client’s nutritional status is improving?

A) Creatinine has dropped from 1.9 to 0.5 mg/dL.
B) Blood urea nitrogen (BUN) level has dropped from 15 to 11 mg/dL.
C) Prealbumin level has risen from 9 to 13 mg/dL.
D) Sodium has risen from 130 to 144 mg/dL.
C

The prealbumin level is a good measure of nutritional status because its half-life is only 2 days, so it reflects current nutritional status. The client’s prealbumin level is rising and almost normal, indicating that the client’s nutritional status is improving. The other laboratory values are more reflective of fluid balance and kidney function.

When conducting a health history assessment, the nurse would want to know what important information about the patient’s elimination status? (Select all that apply.)

A) Time of day patient defecates
B) Patient’s preferences for toileting
C) List of medications taken by patient
D) Recent changes in elimination patterns
E) Changes in color, consistency, or odor of stool or urine
F) Discomfort or pain with elimination
C,D,E,F

Recent changes in elimination patterns, color, consistency, or odor are important for the nurse to know concerning elimination. Discomfort or pain during elimination is important for the nurse to know. A nurse should also know which medications the patient is on as this may affect elimination. Time of day is not important, nor is the patient’s preferences for toileting. They are personal preferences and do not affect elimination.

A confused client is hospitalized for possible pneumonia and is admitted from the emergency department with an indwelling catheter in place. During interdisciplinary rounds the following day, what question by the nurse takes priority?

A) “Can we discontinue the in-dwelling catheter?”
B) “Will the client be able to return home?”
C) “Should we get another chest x-ray today?”
D) “Do you want daily weights on this client?”
A

An in-dwelling catheter dramatically increases the risks of urinary tract infection and urosepsis. Nursing staff should ensure that catheters are left in place only as long as they are medically needed. The nurse should inquire about removing the catheter. All other questions might be appropriate, but because of client safety, this question takes priority.

The nurse is assessing a client who had a stroke in the right cerebral hemisphere. Which neurologic deficit does the nurse assess for in this client?

A) Agraphia
B) Aphasia
C) Impaired olfaction
D) Impaired proprioception
D

A stroke to the right cerebral hemisphere causes impaired visual and spatial awareness. The client may present with impaired proprioception and may be disoriented as to time and place. The right cerebral hemisphere does not control speech, smell, or the client’s ability to write.

A client has newly diagnosed diabetes. To delay the onset of microvascular and macrovascular complications in this client, the nurse stresses that the client take which action?

A) Restrict fluid intake.
B) Prevent ketosis.
C) Control hyperglycemia.
D) Prevent hypoglycemia.
C
Hyperglycemia is a critical factor in the pathogenesis of long-term diabetic complications. Maintaining tight glycemic control will help delay the onset of complications. Preventing hypoglycemia and ketosis, although important, is not as important as maintaining daily glycemic control. Restricting fluid intake is not part of the treatment plan for clients with diabetes.

Which interventions are necessary to provide safe, quality care to a patient receiving enteral tube feedings? SELECT ALL THAT APPLY!!

A) check the residual volume every 4-6 hours
B) use clean technique when changing the feeding system
C) keep the head of the beg elevated at least 30 degrees
D) change the feeding bag & tubing every 12 hours
E) allow closed system containers to hang for 24 hours
A,B,C,E

A client with a pressure ulcer has the following laboratory values: white blood count 8000/mm3, prealbumin 15.2 mg/dL, albumin 4.2 mg/dL, and lymphocyte count 2000/mm3. Which action by the nurse is most appropriate?

A) Request a dietary consult.
B) Assess the client’s vital signs.
C) Document the findings.
D) Place the client in isolation.
A

Albumin, prealbumin, and lymphocyte counts all give information related to nutritional status. The albumin and lymphocyte counts given are normal. The white blood cell count is not directly related to nutritional status. The prealbumin count is low and is a more specific indicator of nutritional status than is the albumin count. This puts the client at risk for impaired wound healing, so the nurse should request a dietary consult.

A nurse is explaining to a student nurse about perfusion. The nurse knows the student understands the concept of perfusion when the student states, “Perfusion

A) is a normal function of the body, and I don’t have to be concerned about it.”
B) varies as a person ages, so I would expect changes in the body.”
C) is monitored by the physician, and I just follow orders.”
D) is monitored by vital signs and capillary refill.”
D

The best method to monitor perfusion is to monitor vital signs and capillary refill. This allows the nurse to know if perfusion is adequate to maintain vital organs. The nurse does have to be concerned about perfusion. Perfusion is not only monitored by the physician but the nurse too. Perfusion does not always change as the person ages.

The nurse is a assessing a client with hypertension. Which client outcome is indicative of effective hypertension management?

A) No complaints of sexual dysfunction occur.
B) Pedal edema is not present in the lower legs.
C) No indication of renal impairment is present.
D) The blood pressure reading is 148/94 mm Hg.
C

One expected outcome for a client with hypertension is for the client to have no evidence of target organ damage, such as renal or heart disease, that can occur with poorly managed hypertension. Development of pedal edema is not directly related to the management of hypertension. Side effects of some hypertensive agents may interfere with sexual function, but this does not relate to the effectiveness of treatment for hypertension. The blood pressure reading is too high to demonstrate effective management.

What statement indicates that the client understands teaching about neutropenia?

A) “I will call my doctor if I have an increase in temperature.”
B) “My grandchildren may get an infection from me.”
C) “I need to use a soft toothbrush.”
D) “I have to wear a mask at all times.”
A

Bone marrow suppression leads to neutropenia and increases the client’s risk for infection. Decreased numbers of neutrophils and other white blood cells can minimize the clinical manifestations of infection. For this reason, the client may not develop a high temperature, even with severe infection, and any elevation of temperature should be reported immediately to the health care provider. The client does not need to wear a mask or use a soft toothbrush (although if the client has low platelets, he or she should use a soft toothbrush to avoid causing trauma). The client is not contagious.

A client has a small-bore nasoenteric feeding tube. The nurse assesses the following vital signs: temperature, 100.2° F (37.8° C); pulse, 112 beats/min; respiratory rate, 22 breaths/min; and blood pressure, 106/62 mm Hg. Which action by the nurse takes priority?

A) Auscultate bowel sounds and slow the feeding down.
B) Remove the tube immediately and notify the heath care provider.
C) Auscultate lung sounds and obtain oxygen saturation.
D) Add blue dye to the feeding tube formula.
C

The client may have aspirated. The nurse should further assess the client’s respiratory and oxygenation status. The client may have another reason for the abnormal vital signs, so the nurse should not pull out the tube before performing other assessments. Adding blue dye to the tube feeding formula is not recommended to check for aspiration. Slowing the feeding down will not be helpful.

A client is receiving a chemotherapeutic agent intravenously through a peripheral line. What is the nurse’s first action when the client reports burning at the site?

A) Apply a cold compress.
B) Discontinue the infusion.
C) Slow the rate of infusion.
D) Check for a blood return.
B

Both irritants and vesicants can cause tissue damage. If the nurse suspects extravasation, he or she should immediately stop the infusion. Even if the IV has a good blood return, some of the chemotherapeutic agent can still be leaking into the tissues. Slowing the rate of infusion is not sufficient to prevent further leakage and damage. Applying a cold compress may or may not be the correct action, depending on the specific agent. However, the compress would be applied only after the infusion has been discontinued.

which statement about a patient with a tube feeding indicates best practice for patient safety & quality care?

A) if the tube becomes clogged, use 30 mL of water for flushing, while applying gentle pressure with a 50 mL piston syringe
B) when administering medications, use cold water to dissolve the drug before administering it
C) use cranberry juice to flush the tube if it is clogged
D) administer drugs down the feeding tube without flushing first, but flush the feeding tube after the drug is given
A

A client has a wound on his left trochanter that is 4 inches in diameter, with black tissue at the perimeter, and bone is exposed. Which is the nurse’s best action?

A) Document as a stage I pressure ulcer and apply a transparent dressing.
B) Document as a stage II pressure ulcer and start wet-to-dry gauze treatments.
C) Document as a stage IV pressure ulcer and prepare the client for débridement.
D) Document as a stage III pressure ulcer and start antibiotic therapy.
C

A stage IV ulcer is one in which skin loss is full thickness, with extensive destruction, tissue necrosis, and/or damage to muscle, bone, or supporting structures. Eschar may be present. When the bone of the trochanter area is visible, tissue loss includes muscle loss. A potential intervention consists of débridement of the necrotic tissue and a possible graft to promote healing.

After initial placement of NG tubes is confirmed, how often must placement be checked? SELECT ALL THAT APPLY?

A) before medication administration
B) it is not necessary to recheck placement
C) every 4-8 hours during feeding
D) before intermittent feeding
E) according to facility policy
A,C,E

The nurse is preparing to administer tube feedings through a client’s new Salem sump nasogastric tube. The nurse is unable to withdraw any fluid from the tube before starting the feeding. Which is the priority action of the nurse?

A) Start the tube feeding as ordered and check the residual in 30 minutes.
B) Inject air into the nasogastric tube while auscultating the client’s epigastric area.
C) Lower the head of the client’s bed and attempt to aspirate fluid again.
D) Obtain orders for a chest x-ray to confirm placement before starting the feeding.
D

The nurse must verify tube placement before beginning any tube feeding or administering any medications through a tube. The most accurate way to determine placement is via chest x-ray. The nurse could cause the client to aspirate if she or he started the feeding then checked later for placement. Insufflation does not provide accurate results and should not be used to verify tube placement. The nurse must keep the client’s head elevated at least 30 degrees.

A client has a urinary tract infection. Which assessment by the nurse is most helpful?

A) Palpating and percussing the kidneys and bladder
B) Performing a bladder scan to assess post-void residual
C) Assessing medical history and current medical problems
D) Inquiring about recent travel to foreign countries
C

Clients who are severely immune compromised or who have diabetes mellitus are more prone to fungal urinary tract infection. The nurse should assess for these factors. A physical examination and a post-void residual may be needed, but not until further information is obtained. Travel to foreign countries probably would not be as important, because even if exposed, the client needs some degree of immune compromise to develop a fungal urinary tract infection.

When a diabetic patient asks about maintaining adequate blood glucose levels, which of the following statements by the nurse relates most directly to the necessity of maintaining blood glucose levels no lower than about 74 mg/dl?

A) “Without a minimum level of glucose circulating in the blood, erythrocytes cannot produce ATP.”
B) “The presence of glucose in the blood counteracts the formation of lactic acid and prevents acidosis.”
C) “The central nervous system cannot store glucose and needs a continuous supply of glucose for fuel.”
D) “Glucose is the only type of fuel used by body cells to produce the energy needed for physiologic activity.”
C

The brain cannot synthesize or store significant amounts of glucose; thus a continuous supply from the body’s circulation is needed to meet the fuel demands of the central nervous system.

The nurse is caring for an overweight client who gained 10 pounds during the previous 2 weeks. The client states that she is hungry all the time and doesn’t understand why. Which assessment finding could explain the client’s weight gain and hunger?

A) The client’s glycosylated hemoglobin level is 6%.
B) The client started taking dexamethasone (Decadron) daily.
C) The client started taking naproxen sodium (Naprosyn) daily.
D) The client’s thyroxine (T4) level is 8 mcg/dL.
B

Dexamethasone is a corticosteroid. These drugs alter carbohydrate, protein, and lipid metabolism, predisposing the client to obesity when taken on a long-term basis. In addition, corticosteroids increase the client’s appetite. Naprosyn is an NSAID, which can lead to gastric upset and ulceration and decreased appetite and weight loss. The client’s glycosylated hemoglobin and thyroid levels are within normal limits and would not explain the hunger and weight gain.

The nurse is prioritizing care to prevent pressure sores for a client who is immobilized. Which interventions are appropriate? (Select all that apply.)

A) Use a rubber ring to decrease sacral pressure when up in the chair.
B) Place a small pillow between bony surfaces.
C) Keep the heels off the bed surfaces.
D) Use a lift sheet to assist with repositioning.
E) Reposition the client who is in a chair every 2 hours.
F) Elevate the head of the bed to 45 degrees.
G) Limit fluids and proteins in the diet.
B,C,D

A small pillow decreases the risk for pressure between bony prominences, a lift sheet decreases friction and shear, and heels have poor circulation and are at high risk for pressure sores, so they should be kept off hard surfaces. Head of the bed elevation greater than 30 degrees increases pressure on pelvic soft tissues. Fluids and proteins are important for maintaining tissue integrity. Clients should be repositioned every hour while sitting in a chair. A rubber ring impairs capillary blood flow, increasing the risk for a pressure sore.

A client who first experienced symptoms related to a confirmed thrombotic stroke 2 hours ago is brought to the intensive care unit. Which prescribed medication does the nurse prepare to administer?

A) Tissue plasminogen activator
B) Heparin sodium
C) Warfarin (Coumadin)
D) Gabapentin (Neurontin)
A

The client who has had a thrombotic stroke has a 3-hour time frame from the onset of symptoms to receive recombinant tissue plasminogen activator (rt-PA) to dissolve the cerebral artery occlusion and re-establish blood flow. Clients must meet eligibility criteria for administration of this therapy. The other medications do not assist in the re-establishment of blood flow for a client with a confirmed thrombotic stroke.

A nurse is about to administer the first dose of captopril (Capoten) to a client with hypertension. Which is the priority nursing intervention?

A) Place the client in Trendelenburg position to facilitate blood flow to the heart.
B) Take the client’s apical pulse for 1 full minute before drug administration.
C) Instruct the client to drink 3 L of fluid daily when taking this medication.
D) Educate the client to sit on the side of the bed for a few minutes before rising.
D

Angiotensin-converting enzyme (ACE) inhibitors such as captopril can cause severe hypotension with initial use. The client should be instructed to rise slowly and sit on the side of the bed for a few minutes to prevent hypotension-induced falls. No indication is known for assessment of the apical pulse for 1 full minute before taking captopril. Placing the client in a Trendelenburg position is not indicated. In case of a precipitous drop in blood pressure, a modified Trendelenburg position may be used. Adequate fluid intake is necessary but is not the priority in this situation.

A client who had a stroke is receiving clopidogrel (Plavix). Which adverse effect does the nurse monitor for in this client?

A) New-onset confusion
B) Repeated syncope
C) Abdominal distention
D) Spontaneous ecchymosis
D

Clopidogrel (Plavix) is an antiplatelet medication that can cause bleeding, bruising, and liver dysfunction. The nurse should be alert for signs of bleeding, such as ecchymosis, bleeding gums, and tarry stools. Plavix does not cause syncope, confusion, or abdominal distention.

A client receiving intravenous chemotherapy asks the nurse the reason for wearing a mask, gloves, and gown while administering drugs to the client. What is the nurse’s best response?

A) “I am preventing the spread of infection from you to me or any other client here.”
B) “The clothing protects me from accidentally absorbing these drugs.”
C) “The policy is for any nurse giving these drugs to wear a gown, gloves, and mask.”
D) “These coverings protect you from getting an infection from me.”
B

Most chemotherapy drugs are absorbed through the skin and mucous membranes. As a result, health care workers who prepare or give these drugs, especially nurses and pharmacists, are at risk for absorbing them. Even at low doses, chronic exposure to chemotherapy drugs can affect health. The Oncology Nursing Society and the Occupational Safety and Health Administration (OSHA) have specific guidelines for using caution and wearing protective clothing whenever preparing, giving, or disposing of chemotherapy drugs.

The earliest and most sensitive assessment finding that would indicate an alteration in intracranial regulation would be:

A) change in level of consciousness.
B) unequal pupil size.
C) loss of primitive reflexes.
D) inability to focus visually.
A

A change in level of consciousness is the earliest and most sensitive indication of a change in intracranial processing. This is assessed with the Glasgow Coma Scale (GCS), which assesses eye opening and verbal and motor response. The inability to focus may indicate a change, but it is not one of the earliest indicators or a component of the GCS. Primitive reflexes refers to those reflexes found in a normal infant that disappear with maturation. These reflexes may reappear with frontal lobe dysfunction and may be tested for with a suspected brain injury, so it would be the reappearance of primitive reflexes. A change in pupil size or unequal pupils may indicate a change, but they are not one of the earliest indicators or a component of the GCS.

Which client statement indicates a good understanding regarding antibiotic therapy for recurrent urinary tract infections?

A) “Even if I feel completely well, I should take the medication until it is gone.”
B) “When my urine no longer burns, I will no longer need to take the antibiotics.”
C) “If my urine becomes lighter and clearer, I can stop taking my medicine.”
D) “If I have a fever higher than 100° F (37.8° C), I should take twice as much medicine.”
A

Antibiotic therapy is most effective, especially for recurrent urinary tract infections, when the client takes the prescribed medication for the entire course, not just when symptoms are present. The other statements demonstrate that additional teaching is needed for the client.

A client presents with a pressure ulcer on the ankle. Which is the first intervention that the nurse implements?

A) Place the client in bed and instruct him or her to elevate the foot.
B) Prepare for and assist with obtaining a wound culture.
C) Assess the affected leg for pulses, skin color, and temperature.
D) Draw blood for albumin, prealbumin, and total protein.
C

A client with an ulcer on the foot should be assessed for interruption in arterial flow to the area. This begins with assessment of pulses and color and temperature of the skin. The nurse can also assess for pulses noninvasively with a Doppler if unable to palpate with his or her fingers. Elevation of the foot would impair the ability of arterial blood to flow to the area. Wound cultures are done after it has been determined drainage, odor, and other risks for infection are present. Tests to determine nutritional status and risk assessment would be completed after the initial assessment is done.

During assessment of a client with a 15-year history of diabetes, the nurse notes that the client has decreased tactile sensation in both feet. Which action does the nurse take first?

A) Notify the health care provider.
B) Document the finding in the client’s chart.
C) Examine the client’s feet for signs of injury.
D) Test sensory perception in the client’s hands.
C

Diabetic neuropathy is common when the disease is of long duration. The client is at great risk for injury in any area with decreased sensation because he or she is less able to feel injurious events. Feet are common locations for neuropathy and injury, so the nurse should inspect them for any signs of injury. After assessing, the nurse should document findings in the client’s chart. Testing sensory perception in the hands may or may not be needed. The health care provider can be notified after assessment and documentation have been completed.

Which nursing intervention best assists a bedridden client to keep skin intact?

A) Use a lift sheet to move the client in bed.
B) Turn the client every 2 to 4 hours.
C) Use a foam mattress pad.
D) Apply talcum powder to the perineal area.
A

Friction forces are generated when the client is dragged or pulled across bed linen; this often leads to altered skin integrity. Using a lift sheet will prevent friction. Keeping the skin clean and dry is an important intervention, but powders should not be used in the perineal area. To minimize vasoconstriction and possible pressure ulcer development from dependency, the client should be turned at a minimum of every 2 hours. A foam mattress will not significantly decrease pressure to an area.

A client presents with an acute exacerbation of multiple sclerosis. Which prescribed medication does the nurse prepare to administer?

A) Interferon beta-1b (Betaseron)
B) Baclofen (Lioresal)
C) Methylprednisolone (Medrol)
D) Dantrolene sodium (Dantrium)
C

Methylprednisolone is the drug of choice for acute exacerbations of the disease. The other medications are not appropriate.

The nurse is assessing a client’s understanding of his hypertension therapy. What client statement indicates a need for further teaching?

A) “When my blood pressure is normal, I will no longer need to take medication.”
B) “If my blood pressure stays under control, I will reduce my risk for a heart attack.”
C) “If I lose weight, I might be able to reduce my blood pressure medication.”
D) “When getting out of bed in the morning, I will sit for a few moments then stand.”
A

Compliance with antihypertensive therapy is difficult for two reasons. First, often clients have no distressing symptoms associated with hypertension and may not believe that they have a problem. Second, many clients believe that once blood pressure is brought back into the normal range, they are “cured” and no longer need to take medication. Losing weight might allow the client to reduce medications. Lowering blood pressure does lower risk for heart attack. Because blood pressure medications often lead to orthostatic hypotension, clients should be taught to change position slowly, sitting first before standing after lying flat.

Which of the following would be included in the assessment of a patient with diabetes mellitus who is experiencing a hypoglycemic reaction? (Select all that apply.)

A) Constricted pupils
B) Flushed skin
C) Tremors
D) Nervousness
E) Extreme thirst
F) Profuse perspiration
C,D,F

When hypoglycemia occurs, blood glucose levels fall, resulting in sympathetic nervous system responses such as tremors, nervousness, and profuse perspiration. Dilated pupils would also occur, not constricted pupils. Extreme thirst, flushed skin, and constricted pupils are consistent with hyperglycemia.

A client who has had a stroke with left-sided hemiparesis has been referred to a rehabilitation center. The client asks, “Why do I need rehabilitation?” How does the nurse respond?

A) “Rehabilitation will reverse any physical deficits caused by the stroke.”
B) “Rehabilitation will help you function at the highest level possible.”
C) “If you do not have rehabilitation, you may never walk again.”
D) “Your doctor knows best and has ordered this treatment for you.”
B

The goal of rehabilitation is to maximize the client’s abilities in all aspects of life. The other responses do not answer the client’s question appropriately.

The nurse is monitoring a client with hypoglycemia. Glucagon provides which function?

A) It enhances the activity of insulin, restoring blood glucose levels to normal more quickly after a high-calorie meal.
B) It prevents hypoglycemia by promoting release of glucose from liver storage sites.
C) It is a storage form of glucose and can be broken down for energy when blood glucose levels are low.
D) It converts excess glucose into glycogen, lowering blood glucose levels in times of excess.
B

Glycogen is a counterregulatory hormone secreted by the alpha cells of the pancreas when blood glucose levels are low. The actions of glycogen that raise blood glucose levels include stimulating the liver to break down glycogen (glycogenolysis) and forming new glucose from protein breakdown (gluconeogenesis). The other statements are not accurate descriptions of the actions of glucagon.

A client has a deep wound covered with a wet-to-damp dressing. Which intervention does the nurse include on this client’s care plan?

A) Apply a new dressing when the seal breaks and the dressing leaks.
B) Change the dressing when the current dressing is saturated.
C) Leave the dressing intact until next week.
D) Change the dressing every 6 hours around the clock.
D

Wet-to-damp dressings are changed every 4 to 6 hours to provide maximum débridement. Synthetic dressings can be left in place for extended periods of time but need to be changed if the seal breaks and the exudate is leaking. Dry gauze dressings should be changed when the outer layer becomes saturated.

A client is hospitalized with a urinary tract infection (UTI). Which clinical manifestation alerts the nurse to the possibility of a complication from the UTI?

A) Hematuria
B) Fever and chills
C) Cloudy, dark urine
D) Burning on urination
B

Lower urinary tract infections are rarely associated with systemic symptoms of fever and chills. A client with a UTI who develops fever and chills should be assessed for the development of pyelonephritis. The other options can be seen with UTI.

The nurse observes a small opening that is draining purulent material on the skin over the trochanter area of a bedridden client. Which is the nurse’s next best action?

A) Probe for a larger pocket of necrotic tissue.
B) Apply alginate dressing daily.
C) Apply a transparent film dressing.
D) Measure the reddened area on the skin surface.
A

This “hidden” wound may first be observed as a small opening in the skin through which purulent drainage exudes. Applying a transparent film dressing would not help this type of wound to heal. Measuring the reddened area would not assist in determining the actual size of the wound, because internal damage has occurred. Alginate dressings could not be applied if the area were not opened.

When reviewing an older client’s medical record, which findings lead the nurse to perform a nutrition assessment? (Select all that apply.)

A) Widow/widower status
B) Chronic constipation
C) Cholecystectomy 4 years ago
D) Random blood sugar level of 198 mg/dL
E) History of depression
F) Inability to afford a new pair of glasses
A,B,E,F

Many factors contribute to malnutrition in older clients. Depression and loneliness from the loss of a spouse; constipation; poor eyesight; chronic medical problems, including depression; and taking prescription and/or over-the-counter medications can contribute to malnutrition. Blood glucose levels and a previous cholecystectomy would not necessarily contribute.

The nurse is caring for a female client who is 5 feet, 7 inches tall and weighs 115 pounds. The client asks the nurse if she needs to lose weight. Which response by the nurse is best?

A) “No. In fact, your body mass index suggests that you are already underweight.”
B) “Yes. Your body mass index suggests you are slightly overweight.”
C) “Your weight is just fine. Don’t worry about it.”
D) “Maybe. Let’s look at your risks for cardiovascular disease.”
A

The client’s body mass index (BMI) is 18.0, so she is already underweight. It is inaccurate to tell the client she is overweight, and it is unnecessary to consider her weight in light of any cardiovascular risk factors. The nurse should not reassure the client that her weight is just fine because she is underweight.

A client who has type 2 diabetes is prescribed glipizide (Glucotrol). Which precautions does the nurse include in the teaching plan related to this medication?

A) “Avoid taking nonsteroidal anti-inflammatory drugs.”
B) “Change positions slowly when you get up.”
C) “If you miss a dose of this drug, you can double the next dose.”
D) “Discontinue the medication if you develop an infection.”
A

Nonsteroidal anti-inflammatory drugs potentiate the hypoglycemic effects of sulfonylurea agents. Glipizide is a sulfonylurea. The other statements are not applicable to glipizide.

The nurse is caring for a client who had a stroke. Which nursing intervention does the nurse implement during the first 72 hours to prevent complications?

A) Position with the head of the bed flat to enhance cerebral perfusion.
B) Monitor neurologic and vital signs closely to identify early changes in status.
C) Administer prescribed analgesics to promote pain relief.
D) Cluster nursing procedures together to avoid fatiguing the client.
B

Early detection of neurologic, blood pressure, and heart rhythm changes offers an opportunity to intervene in a timely fashion. Evidence is not yet sufficient to recommend a specific back rest elevation after stroke. Analgesics are often held during the first 72 hours to ensure that the client’s neurologic status is not altered by pain medications. Preventing fatigue is not a priority in the first 72 hours.

A client with diabetes is prescribed insulin glargine once daily and regular insulin four times daily. One dose of regular insulin is scheduled at the same time as the glargine. How does the nurse instruct the client to administer the two doses of insulin?

A) “Draw up and inject the insulin glargine first, then draw up and inject the regular insulin.”
B) “First draw up the dose of regular insulin, then draw up the dose of insulin glargine in the same syringe, mix, and inject the two insulins together.”
C) “First draw up the dose of insulin glargine, then draw up the dose of regular insulin in the same syringe, mix, and inject the two insulins together.”
D) “Draw up and inject the insulin glargine first, wait 20 minutes, then draw up and inject the regular insulin.”
A

Insulin glargine must not be diluted or mixed with any other insulin or solution. Mixing results in an unpredictable alteration in the onset of action and time to peak action. The correct instruction is to draw up and inject first the glargine, then the regular insulin right afterward.

The nurse would identify which body systems as directly involved in the process of normal gas exchange? (Select all that apply.)

A) Endocrine system
B) Neurologic system
C) Hepatic system
D) Immune system
E) Cardiovascular system
F) Pulmonary system
B, E, F

The neurologic system controls respiratory drive; the respiratory system controls delivery of oxygen to the lung capillaries; and the cardiac system is responsible for the perfusion of vital organs. These systems are primarily responsible for the adequacy of gas exchange in the body. The endocrine and hepatic systems are not directly involved with gas exchange. The immune system primarily protects the body against infection.

The nurse is discharging home a client at risk for venous thromboembolism (VTE) on low-molecular-weight heparin. What instruction does the nurse provide to this client?

A) “You must have your aPTT checked every 2 weeks.”
B) “Notify your health care provider if your stools appear tarry.”
C) “An IV catheter will be placed to administer your heparin.”
D) “Massage the injection site after the heparin is injected.”
B

As with any anticoagulation, low-molecular-weight heparin incurs risk of bleeding. Clients should be taught to report to their health care provider the presence of tarry stools, bleeding gums, hematuria, ecchymosis, or petechiae. Low-molecular-weight heparin does not affect activated partial thromboplastin time (aPTT), as does intravenous heparin. This type of heparin is administered subcutaneously to deliver a slow sustained response. Massaging the site would hasten absorption and decrease effects.

Which is the highest priority goal to set for a client with pneumonia?

A) Maintenance of SaO2 of 95%
B) Walking 20 feet three times daily
C) Absence of cyanosis
D) Absence of confusion
A

Maintenance of an SaO2 of at least 95% is a clear goal that indicates that the client has adequate oxygenation. Absence of cyanosis and the presence of confusion are assessment factors that contribute to evaluation of oxygen; however, they are not absolute measures. Likewise, walking three times a day does not directly address oxygenation.

The nurse is assessing a client admitted to the cardiac unit. What statement made by the client alerts the nurse to the possibility of right-sided heart failure?

A) “I sleep with four pillows at night.”
B) “I wake up coughing every night.”
C) “My shoes fit really tight lately.”
D) “I have trouble catching my breath.”
C

Signs of systemic congestion occur with right-sided heart failure. Fluid is retained, pressure builds in the venous system, and peripheral edema develops. Left-sided heart failure symptoms include respiratory symptoms. Orthopnea, coughing, and difficulty breathing all could be results of left-sided heart failure.

The nurse is caring for a client with Crohn’s disease and colonic strictures. Which assessment finding requires the nurse to consult the health care provider immediately?

A) Traces of blood in the stool
B) Distended abdomen
C) Temperature of 100.0° F (37.8° C)
D) Crampy lower abdominal pain
B

The presence of strictures predisposes the client to intestinal obstruction. Abdominal distention may indicate that the client has developed an obstruction of the large bowel, and the client’s provider should be notified right away. Low-grade fever, bloody diarrhea, and crampy abdominal pain are common symptoms of Crohn’s disease.

What is my favorite football team?

A) I hate football!!
B) the Fightin’ Irish
C) GO BUCKS!!!!
D) that team up north
C

Seriously, if you don’t know the instructor’s favorite football by this time, you need to listen to your recordings from the first lecture. Just sayin’

The nurse is caring for a client who is receiving heparin therapy for a venous thromboembolism (VTE). The client’s activated partial thromboplastin time (aPTT) before heparin therapy was 30 seconds. Which aPTT result indicates that anticoagulation is adequate at this time?

A) 30 seconds
B) 150 seconds
C) 60 seconds
D) 15 seconds
C

Therapeutic aPTT values for clients receiving heparin should range from 1.5 to 2.5 times the control value.

The nurse is caring for a patient who is being discharged home after a splenectomy. What information on immune function needs to be included in this patient’s discharge planning?

A) Limiting contact with the general population
B) The importance of wearing a face mask in public
C) The mechanisms of the inflammatory response
D) Basic infection control techniques
D

The spleen is one of the major organs of the immune system. Without the spleen, the patient is at higher risk for infection; so, the nurse must be sure that the patient understands basic principles of infection control. The patient with a splenectomy does not need to understand the mechanisms of inflammatory response. The patient with a splenectomy does not need to wear a face mask in public as long as the patient understands and maintains the basic principles of infection control. The patient who has had a splenectomy does not need to limit contact with the general population as long as the patient understands and maintains the basic principles of infection control.

A client is admitted with infection and a high fever. Which assessments by the nurse take priority? (Select all that apply.)

A) Skin turgor
B) Pulse quality
C) Blood pressure
D) Bowel sounds
E) Respiratory effort
F) Mental status
A, B, C, F
Dehydration can accompany fever, especially if the client is sweating profusely. Blood pressure, pulse quality, and skin turgor are assessments of fluid status. Mental status changes can accompany fluid losses, especially in older clients.

The health care provider has prescribed a client sodium warfarin (Coumadin) while he is still receiving intravenous heparin. Which is the nurse’s best action?

A) Turn off the heparin before administering the warfarin.
B) Clarify the warfarin order with the nursing supervisor.
C) Administer both heparin and warfarin as prescribed.
D) Hold the warfarin dose until the heparin is discontinued.
C

Although both heparin and warfarin are anticoagulants, they have different mechanisms of action and onsets of action. Because warfarin has such a slow onset, it must be started while the client is still receiving heparin. Once the warfarin is therapeutic, as evidenced by the international normalized ratio (INR), the client’s heparin can be safely discontinued. Effects of heparin will be cleared from the client’s bloodstream within a few hours.

Individuals of low socioeconomic status are at an increased risk for infection because of which of the following? (Select all that apply.)

A) High cost of medications
B) Inadequate nutrition
C) Easy access to health screenings
D) Uninsured or underinsured status
A, B, D

Individuals of low socioeconomic status tend to be part of the underinsured or uninsured population. Lack of insurance decreases accessibility to health care in general and health screening services specifically. High costs of medication and nutritious food also make this population at higher risk for infection.

The nurse is working on a plan of care with her patient which includes turning and positioning and adequate nutrition to help the patient maintain intact skin integrity. The nurse helps the patient to realize that this breaks the chain of infection by eliminating a:

A) portal of entry.
B) host.
C) mode of transmission.
D) reservoir.
A

Broken or impaired skin creates a portal of entry for pathogens. By maintaining intact tissue, the patient and the nurse have broken the chain of infection by eliminating a portal of entry. Host is incorrect because you are not eliminating the person or organism. Intact tissue does not eliminate the mode of transmission. Skin can still be used to transfer pathogens regardless of it being intact or broken. Intact skin does not eliminate the location for pathogens to live and grow.

The nurse is preparing to administer a prescribed IV antibiotic to a client admitted with a serious infection. Which action by the nurse is most important?

A) Double check the “five rights.”
B) Assess the client for allergies.
C) Teach the client about the drug.
D) Check the IV for patency.
B

All actions are appropriate and important before administering any medications. However, client safety is the priority. The nurse should first assess the client for medication allergies by asking the client or checking the chart (or both). Ensuring a patent IV and checking the five rights will not protect the client from an allergic reaction.

The nurse assesses a cut that is 24 hours old and finds that the site is swollen, red, and tender to the touch. Which cell types are responsible for these assessment findings?

A) Natural killer cells
B) Basophils and eosinophils
C) Erythrocytes and platelets
D) Plasma cells and B-lymphocytes
B

Basophils and eosinophils release histamine, kinins, and other substances that cause the manifestations of inflammation. Erythrocytes carry oxygen, and platelets help stop bleeding. Plasma cells and B-lymphocytes produce antibodies to help fight infection, and natural killer cells destroy invading bacteria.

A female client is admitted with an exacerbation of ulcerative colitis. Which laboratory value does the nurse correlate with this condition?

A) Erythrocyte sedimentation rate (ESR), 55 mm/hr
B) Potassium, 5.5 mEq/L
C) Sodium, 144 mEq/L
D) Hemoglobin, 14.2 g/dL
A

The erythrocyte sedimentation rate (ESR) is an indicator of inflammation, which is elevated during an exacerbation of ulcerative colitis. The normal range for the ESR is 0 to 33 mm/hr. Diarrhea caused by ulcerative colitis will result in loss of potassium and hypokalemia with levels lower than 3.5 mEq/L. Bloody diarrhea will lead to anemia, with hemoglobin levels lower than 12 g/dL in females. The sodium level is normal.

O-H-

A) who cares!
B) I cannot pick this so I will lose points (this is for the Michigan Fans!!)
C) I-O
D) O-No!!
C

Sucking up here with C may be dependent on how low your accumulative test scores are…

A patient is being treated with an antibiotic. The nurse explains to the patient that this medication is required for the reduction of inflammation at the injury site because this medication:

A) will decrease the pain at the site.
B) helps to kill the infection causing the inflammation.
C) will reduce the patients fever.
D) inhibits cyclooxygenase.
B

Antimicrobials treat the underlying cause of the infection which leads to inflammation. Analgesics and nonsteroidal antiinflammatory drugs (NSAIDs) help to treat pain. NSAIDs and other antipyretics are cyclooxygenase inhibitors. Antipyretics help to reduce fever.

The nurse is caring for a client with ulcerative colitis and severe diarrhea. Which nursing assessment is the highest priority?

A) Skin integrity
B) Blood pressure
C) Heart rate and rhythm
D) Abdominal percussion
C

Although the client with severe diarrhea may experience skin irritation and hypovolemia, the client is most at risk for cardiac dysrhythmias secondary to potassium and magnesium loss from severe diarrhea. The client should have her or his electrolyte levels monitored, and electrolyte replacement may be necessary. Abdominal percussion is an important part of physical assessment but has lower priority for this client than heart rate and rhythm.

The nurse assesses a client with pneumonia and notes decreased lung sounds on the left side and decreased lung expansion. What is the nurse’s best action?

A) Increase oxygen flow to 10 L/min.
B) Perform an arterial blood gas analysis.
C) Have the client cough and deep breathe.
D) Check oxygen saturation and notify the health care provider.
D

Decreased lung sounds and decreased lung expansion could indicate the development of a complication such as empyema or pus in the pleural space. The nurse should check the client’s oxygen saturation and notify the provider. Infection can also move into the bloodstream and result in sepsis, so quick treatment is needed.

The nurse is assigned a group of patients. Which patient would the nurse identify as being at increased risk for impaired gas exchange? A patient:

A) with a hemoglobin of 8.5 g/dL
B) with a blood glucose of 350 mg/dL
C) who has been on anticoagulants for 10 days
D) with a heart rate of 100 beats/min and blood pressure of 100/60
A

The hemoglobin is low (anemia), therefore the ability of the blood to carry oxygen is decreased. High blood glucose and/or anticoagulants do not alter the oxygen carrying capacity of the blood. A heart rate of 100 beats/min and blood pressure of 100/60 are not indicative of oxygen carrying capacity of the blood.

The nurse is starting a client on digoxin (Lanoxin) therapy. What intervention is essential to teach this client?

A) “Increase your intake of foods high in potassium.”
B) “Avoid taking aspirin or aspirin-containing products.”
C) “Hold this medication if your pulse rate is below 80 beats/min.”
D) “Do not take this medication within 1 hour of taking an antacid.”
D

Gastrointestinal absorption of digoxin is erratic. Many medications, especially antacids, interfere with its absorption. Clients are taught to hold their digoxin for bradycardia; a heart rate of 80 is too high for this cutoff.

The nurse is caring for a client with Crohn’s disease who has developed a fistula. Which nursing intervention is the highest priority?

A) Position the client to allow gravity drainage of the fistula.
B) Check and record blood glucose levels every 6 hours.
C) Encourage the client to consume a diet high in protein and calories.
D) Monitor the client’s hematocrit and hemoglobin.
C

The client with Crohn’s disease is already at risk for malabsorption and malnutrition. Malnutrition impairs healing of the fistula and immune responses. Therefore, maintaining adequate nutrition is a priority for this client. The client will require 3000 calories per day to promote healing of the fistula. Monitoring the client’s blood sugar and hemoglobin levels is important, but less so than encouraging nutritional intake. The client need not be positioned to facilitate gravity drainage of the fistula, because fistulas often are found in the abdominal cavity.

The nurse is teaching a client who has recently given birth about immunity that has been passed to the newborn. Which statement by the client indicates that additional teaching is needed?

A) “My baby received some antibodies from me before birth, and I will give him more when I breast-feed.”
B) “I had the measles, so my baby will be protected against it until he is old enough to receive the MMR vaccine.”
C) “Only certain antibodies were able to cross the placenta to protect my baby.”
D) “I had chickenpox and am immune to it, so my baby will not need to have the chickenpox vaccine.”
D

The baby receives passive immunity from antibodies that are passed through the placenta in utero. Maternal passive immunity is temporary and will last for only a short time after birth.

The nurse notes a venous ulcer on the client’s left ankle. What additional assessment finding does the nurse expect in this client?

A) Absence of hair on the left lower extremity
B) Skin surrounding the ulcer mottled but blanchable
C) Brownish discoloration of the lower extremity
D) Cold and gray-blue lower extremity
C

Venous ulcers are characterized by brown pigmentation of the skin of the lower extremity. Mottled skin, the presence of dependent rubor, and cyanosis are features of arterial ulcers.

A client has recently been placed on prednisone (Deltasone). What is the highest priority instruction the nurse will provide?

A) “Take the drug with food or milk.”
B) “Report any abdominal pain or dark-colored vomit.”
C) “Expect to experience weight gain.”
D) “Watch your diet while on this medication.”
B

All of these directions are appropriate to give the client; however, telling the client to report abdominal pain and dark-colored vomit is most important because these could signal gastric ulceration.

An older adult client is admitted with an infection. On assessment, the nurse finds the client slightly confused. Vital signs are as follows: temperature 99.2° F (37.3° C), blood pressure 100/60 mm Hg, pulse 100, and respiratory rate 20. Which action by the nurse is most appropriate?

A) Document the findings and continue to monitor.
B) Assess the client’s pain level and treat if needed.
C) Perform a Mini-Mental Status Examination.
D) Assess the client for other signs of infection.
D

Because of an age-related decline in immune function, an older adult’s normal temperature may be 1° to 2° lower than normal. A temperature of 99.2° F may be a fever in this population. Often a change in mental status is an early sign of illness for the older adult. The nurse should assess for other indications of infection.

The nurse is caring for an older postoperative client. Which assessment finding causes the nurse to assess further for a wound infection?

A) The client is now confused but was not confused previously.
B) Moderate serosanguineous drainage is seen on the dressing.
C) The white blood cell count is 8000/mm3.
D) The white blood cell differential indicates a right shift.
A

Older adult clients often do not demonstrate typical signs and symptoms of infection because of the diminished immune function seen with aging. Often, the first sign of infection is mental status changes. Any change in mental status in the older postoperative client should lead the nurse to assess for a wound infection.

The nurse is assessing a client’s skin for local signs of infection. Which signs does the nurse assess for? (Select all that apply.)

A) Redness
B) Fever
C) Increased erythrocyte sedimentation rate (ESR)
D) Pain
E) Swelling
F) Warmth
A,D,E,F

Localized signs of infection include redness, warmth, pain, swelling, heat, and pus. Fever and increased ESR are systemic signs of infection.

Which client is at highest risk of compromised immunity?

A) Client with extreme anxiety
B) Client who is awaiting surgery
C) Client who has just had surgery
D) Client who just delivered a baby
C

Intact skin is a defense to prevent infection; however, a client who has recently had surgery has a portal for organisms to enter the body and cause infection.

A client is admitted with left lower lung pneumonia. Which assessment finding does the nurse correlate with this condition?

A) Expiratory wheeze on the right side
B) Crackles heard on expiration bilaterally
C) Dullness to percussion on the lower left side
D) Crepitus of the skin around the left lung
C

The client with pneumonia may have dullness to percussion on the affected side. The other options are all inconsistent with pneumonia.

The nurse auscultates the following lung sound in the client with pneumonia. What is the best intervention?

Audio Clip
A) Administer IV fluids.
B) Have the client use an incentive spirometer.
C) Have the client cough and deep breathe.
D) Prepare to administer a bronchodilator.
B

The sound heard is crackles. Crackles often indicate atelectasis, which can be reversed by using an incentive spirometer. If no spirometer is available, coughing and deep breathing is the next best option. This client does not have wheezing, so bronchodilators are not indicated. IV fluids would not help atelectasis.

A client with a history of heart failure is being discharged. Which priority instruction will assist the client in the prevention of complications associated with heart failure?

A) “Eat six small meals daily instead of three larger meals.”
B) “When you feel short of breath, take an additional diuretic.”
C) “Avoid drinking more than 3 quarts of liquids each day.”
D) “Weigh yourself daily while wearing the same amount of clothing.”
D

Clients with heart failure are instructed to weigh themselves daily to detect worsening heart failure early, and thus avoid complications. Other signs of worsening heart failure include increasing dyspnea, exercise intolerance, cold symptoms, and nocturia.

The nurse assesses a client’s legs. Which assessment finding indicates arterial insufficiency?

A) Pain with activity but not while resting
B) Dependent mottling and absence of hair
C) Full veins present in dependent extremity
D) Ankle discoloration and pitting edema
B

Dependent mottling and absence of hair is an indication of arterial insufficiency. Pain may be present with activity and at rest. Edema and ankle discoloration would be indicative of venous insufficiency.

The nurse is teaching a client with pneumonia ways to clear secretions. Which intervention is the most effective?

A) Administering an antiemetic medication
B) Increasing fluids to 2 L/day if tolerated
C) Administering an antitussive medication
D) Having the client cough and deep breathe hourly
B

Increasing fluids has been proven to decrease the thickness of secretions, thus allowing them to be expectorated quickly. The other interventions would not be as effective.

The nurse is assessing a client with left-sided heart failure. What conditions does the nurse assess for? (Select all that apply.)

A) S3/S4 summation gallop
B) Cough worsens at night
C) Dependent edema
D) Pulmonary crackles
E) Confusion, restlessness
F) Pulmonary hypertension
A,B,D,E
Left-sided failure occurs with a decrease in contractility of the heart or an increase in afterload. Most of the signs will be noted in the respiratory system. Right-sided failure occurs with problems from the pulmonary vasculature onward. Signs will be noted before the right atrium or ventricle.

The nurse assesses the patient and notes all of the following. Select all of the findings that indicate the systemic manifestations of inflammation.

A) Oral temperature 38.6 F
B) WBC 20
C) Thick, green nasal discharge
D) Patient reports, “I’m tired all the time. I haven’t felt like myself in days”
E) Patient complaint of pain at 6 on a 0 to 10 scale on palpation of frontal and maxillary sinuses
A,B,D

Systemic manifestations of inflammatory response include elevated temperature, leukocytosis, and malaise and fatigue. Purulent exudates and pain are both considered local manifestations of inflammation.

A client with chronic obstructive pulmonary disease (COPD) reports social isolation. What does the nurse encourage the client to do?

A) Participate in community activities.
B) Verbalize his or her thoughts and feelings.
C) Ask the client’s physician for an antianxiety agent.
D) Join a support group for people with COPD.
B

Many clients with moderate to severe COPD become socially isolated because they are embarrassed by frequent coughing and mucus production. They also can experience fatigue, which limits their activities. The nurse needs to encourage the client to verbalize thoughts and feelings so that appropriate interventions can be selected. Joining a support group would not decrease feelings of social isolation if the client does not verbalize feelings. Antianxiety agents will not help the client with social isolation. Encouraging a client to participate in activities without verbalizing concerns also would not be an effective strategy for decreasing social isolation.

The nurse is assessing a client with lung disease. Which symptom does the nurse intervene for first?

A) The client’s anterior-posterior chest diameter is 2:2.
B) Clubbing of the finger tips is noted.
C) The client is pale.
D) The client has bilateral dependent leg edema.
D

The client with bilateral dependent edema may be developing right-sided heart failure in response to respiratory disease. This symptom should be investigated right away and reported to the health care provider. Further assessment is needed. The client with chronic lung disease may develop increased anterior-posterior diameter and clubbing as responses to chronic hypoxia. These symptoms do not require immediate intervention. The client is often pale or has a dusky appearance; this also would not warrant immediate intervention.

A patient is diagnosed with a sprain to her right ankle after a fall. The patient asks the nurse about using ice on her injured ankle. The nurse should tell the patient that:

A) ice is not recommended for use on the sprain because it would inhibit the inflammatory response.
B) ice should be applied for 15 to 20 minutes every 2 to 3 hours over the next 1 to 2 days.
C) she should use ice only when the ankle hurts.
D) she should wrap an ice pack around the injured ankle for the next 24 to 48 hours.
B

Ice is used on areas of injury during the first 24 to 48 hours after the injury occurs to prevent damage to surrounding tissues from excessive inflammation. Ice should be used for a maximum of 20 minutes at a time every 2 to 3 hours. Ice must be used according to a schedule for it to be effective and not be overused. Using ice more often or for longer periods of time can cause additional tissue damage. Ice is recommended to inhibit the inflammatory process from damaging surrounding tissue.

The nurse is assessing a patient for the adequacy of ventilation. What assessment findings would indicate the patient has good ventilation? (Select all that apply.)

A) There is presence of quiet, effortless breath sounds at lung base bilaterally.
B) Nail beds are pink with good capillary refill.
C) Trachea is just to the left of the sternal notch.
D) Respiratory rate is 24 breaths/min.
E) The right side of the thorax expands slightly more than the left.
F) Oxygen saturation level is 98%.
A,B,F

Oxygen saturation level should be between 95 and 100%; nail beds should be pink with capillary refill of about 3 seconds; and breath sounds should be present at base of both lungs. Normal respiratory rate is between 12 and 20 breaths/min. The trachea should be in midline with the sternal notch. The thorax should expand equally on both sides.

The nurse is assessing clients on a cardiac unit. Which client does the nurse assess most carefully for developing left-sided heart failure?

A) Middle-aged woman with aortic stenosis
B) Older woman who smokes cigarettes daily
C) Older man who has had a myocardial infarction
D) Middle-aged man with pulmonary hypertension
A

Although most people with heart failure will have failure that progresses from left to right, it is possible to have left-sided failure alone for a short period. It is also possible to have heart failure that progresses from right to left. Causes of left ventricular failure include mitral or aortic valve disease, coronary artery disease (CAD), and hypertension.

A client asks the nurse why it is important to be weighed every day if he has right-sided heart failure. What is the nurse’s best response?

A) “The hospital requires that all inpatients be weighed daily.”
B) “Weight is the best indication that you are gaining or losing fluid.”
C) “You need to lose weight to decrease the incidence of heart failure.”
D) “Daily weights will help us make sure that you’re eating properly.”
B

Daily weights are needed to document fluid retention or fluid loss. One liter of fluid equals 2.2 pounds.

A patient states that his/her legs have pain with walking that decreases with rest. The nurse observes absence of hair on the patient’s lower leg and the patient has a thready posterior tibial pulse. How would the nurse position the patient’s legs?

A) Slightly bent with a pillow under the knees
B) Dependent position
C) Elevated
D) Crossed at the knee
B

A patient with arterial insufficiency is taught to position their legs in a dependent position to use gravity to help perfuse the tissues. Crossing legs at the knee may interfere with blood flow. Slightly bent legs do not enhance blood flow.

The nurse is caring for a client with peripheral arterial disease. What priority nursing intervention does the nurse perform to promote vasodilation?

A) Increase the client’s exercise regimen daily.
B) Educate the client to abstain from smoking.
C) Apply a heating pad to the affected limb.
D) Administer an aspirin on a daily basis.
B

Smoking causes vasoconstriction, and its effects can last up to 1 hour after the cigarette is finished. Increasing activity may lead to collateral circulation but does not cause vasodilation. Use of a heating pad is contraindicated in the client with peripheral artery disease because of the risk of a burn caused by diminished sensation. The use of aspirin my impede platelet clumping and is contraindicated only when the client is on anticoagulants.

Which statement by a patient indicates additional teaching is required about the medication warfarin?

A) “I will increase the intake of green, leafy vegetables for a more healthful diet.”
B) “I will restrict the intake of foods high in vitamin C.”
C) “I will increase the amount of protein in my diet to protect my kidneys.”
D) “I will continue my diabetic diet and restrict sugar.”
A

Foods such as green, leafy vegetables have high levels of vitamin K. Warfarin is an anticoagulant that acts by interfering with vitamin K-dependent clotting factors. If the amount of vitamin K is increased in the diet, the medication dose may need to be adjusted. A diabetic diet would be continued as indicated for a patient receiving warfarin. Vitamin C is not related to warfarin.

A client has been admitted to the intensive care unit with worsening pulmonary manifestations of heart failure. What is the nurse’s best action?

A) Administer loop diuretics as prescribed.
B) Begin cardiopulmonary resuscitation (CPR).
C) Promote rest and minimize activities.
D) Place the client in a high Fowler’s position.
A

The client with worsening heart failure is most at risk for pulmonary edema as a consequence of fluid retention. Administering diuretics will decrease the fluid overload, thereby decreasing the incidence of pulmonary edema. High Fowler’s position might help the client breathe easier but will not solve the problem. CPR is not warranted in this situation. Rest is important for clients with heart failure, but this is not the priority.

What information about nutrition does the nurse teach a client with chronic obstructive pulmonary disease (COPD)? (Select all that apply.)

A) “Eat dry foods rather than wet foods, which are heavier.”
B) “Increase carbohydrate intake for energy.”
C) “Have about six small meals a day.”
D) “Practice diaphragmatic breathing against resistance four times daily.”
E) “Avoid drinking fluids just before and during meals.”
F) “Eat high-fiber foods to promote gastric emptying.”
G) “Rest before meals if you have dyspnea.”
C, E, G

Fluids can make a client feel bloated and should be avoided with meals. Resting before the meal will help a client with dyspnea. Six small meals a day also will help to decrease bloating. Dry foods can cause coughing. Fibrous foods can produce gas, which can cause abdominal bloating and can increase shortness of breath. Diaphragmatic breathing will not necessarily help nutrition.

Which person is at greatest risk for developing a community-acquired pneumonia?

A) Young adult aerobics instructor who is a vegetarian
B) Middle-aged teacher who typically eats a diet of Asian foods
C) Older adult who smokes and has a substance abuse problem
D) Older adult with exercise-induced wheezing
C

Although age is a factor in the development of community-acquired pneumonia, other lifestyle and exposure factors increase the risk to a greater extent than age. Two conditions that heavily predispose to the development of pneumonia are cigarette smoking and alcoholism. Dietary choices typically do not predispose to the development of pneumonia. Cigarette smoking interferes with the ciliary function of removal of invasive materials. Alcoholism usually results in unbalanced nutrition, as well as decreased immune function. A middle-aged adult, an older adult with wheezing induced by exercise, and a young adult vegetarian would not be at risk for community-acquired pneumonia because they have no predisposing conditions.

In order to provide the best intervention for a patient, the nurse is often responsible for obtaining a sample of exudate for culture. This test will identify:

A) what cells are being utilized by the body to attack an infection.
B) whether a patient has an infection.
C) where an infection is located.
D) what specific type of pathogen is causing an infection.
D

People can transmit pathogens even if they don’t currently feel ill. Some carriers never experience the full symptoms of a pathogen. A CBC will identify that the patient has an infection. Inspection and radiography will help identify where an infection is located. The CBC with differential will identify the white blood cells being used by the body to fight an infection. The culture will grow the microorganisms in the sample for identification of the specific type of pathogen.

The nurse is caring for a client who is experiencing excessive bleeding after receiving unfractionated heparin. What orders does the nurse anticipate from the health care provider? (Select all that apply.)

A) Laboratory draw for prothrombin time (PT)/international normalized ratio (INR)
B) Laboratory draw for activated partial thromboplastin time (aPTT)
C) Administer enoxaparin (Lovenox)
D) Administer protamine sulfate
E) Administer vitamin K
B,D

Protamine sulfate is the antidote for heparin-induced bleeding. Vitamin K is the antidote for warfarin. Warfarin (Coumadin) would increase bleeding. Enoxaparin is another name for heparin.

A client with heart failure is prescribed enalapril (Vasotec). What is the nurse’s priority teaching for this client?

A) “Avoid using aspirin-containing products.”
B) “Take your medication with food.”
C) “Check your pulse daily.”
D) “Avoid using salt substitutes.”
D

Angiotensin-converting enzyme (ACE) inhibitors inhibit the excretion of potassium. Hyperkalemia can be a life-threatening side effect, and clients should be taught to limit potassium intake. Salt substitutes are composed of potassium chloride.

The nurse is assessing a client who reports claudication after walking a distance of one block. The nurse notes a painful ulcer on the fourth toe of the client’s right foot. What condition do these findings correlate with?

A) Peripheral arterial disease
B) Deep vein thrombosis
C) Diabetic foot ulceration
D) Peripheral venous disease
A

Arterial disease is characterized by claudication after walking short distances. Ulcerations caused by peripheral arterial disease are painful and initially are located at the most distal points on the extremity. Diabetic ulcers and venous ulcers are seldom painful and usually tend to occur where pressure is applied.

Following surgery, Mario complains of mild incisional pain while performing deep- breathing and coughing exercises. The nurse’s best response would be:
A. “Pain will become less each day.”
B. “This is a normal reaction after surgery.”
C. “With a pillow, apply pressure against the incision.”
D. “I will give you the pain medication the physician ordered.”
Answer: (C) “With a pillow, apply pressure against the incision.”
Applying pressure against the incision with a pillow will help lessen the intra-abdominal pressure created by coughing which causes tension on the incision that leads to pain.

The nurse needs to carefully assess the complaint of pain of the elderly because older people
A. are expected to experience chronic pain
B. have a decreased pain threshold
C. experience reduced sensory perception
D. have altered mental function
Answer: (C) experience reduced sensory perception
Degenerative changes occur in the elderly. The response to pain in the elderly maybe lessened because of reduced acuity of touch, alterations in neural pathways and diminished processing of sensory data.

Mary received AtropineSO4 as a pre-medication 30 minutes ago and is now complaining of dry mouth and her PR is higher, than before the medication was administered. The nurse’s best
A. The patient is having an allergic reaction to the drug.
B. The patient needs a higher dose of this drug
C. This is normal side-effect of AtSO4
D. The patient is anxious about upcoming surgery
Answer: (C) This is normal side-effect of AtSO4
Atropine sulfate is a vagolytic drug that decreases oropharyngeal secretions and increases the heart rate.

Ana’s postoperative vital signs are a blood pressure of 80/50 mm Hg, a pulse of 140, and respirations of 32. Suspecting shock, which of the following orders would the nurse question?
A. Put the client in modified Trendelenberg’s position.
B. Administer oxygen at 100%.
C. Monitor urine output every hour.
D. Administer Demerol 50mg IM q4h
Answer: (D) Administer Demerol 50mg IM q4h
Administering Demerol, which is a narcotic analgesic, can depress respiratory and cardiac function and thus not given to a patient in shock. What is needed is promotion for adequate oxygenation and perfusion. All the other interventions can be expected to be done by the nurse.

Mr. Pablo, diagnosed with Bladder Cancer, is scheduled for a cystectomy with the creation of an ileal conduit in the morning. He is wringing his hands and pacing the floor when the nurse enters his room. What is the best approach?
A. “Good evening, Mr. Pablo. Wasn’t it a pleasant day, today?”
B. “Mr, Pablo, you must be so worried, I’ll leave you alone with your thoughts.
C. “Mr. Pablo, you’ll wear out the hospital floors and yourself at this rate.”
D. “Mr. Pablo, you appear anxious to me. How are you feeling about tomorrow’s surgery?”
Answer: (D) “Mr. Pablo, you appear anxious to me. How are you feeling about tomorrow’s surgery?”
The client is showing signs of anxiety reaction to a stressful event. Recognizing the client’s anxiety conveys acceptance of his behavior and will allow for verbalization of feelings and concerns.

Med Surg Exam 2023 1 1. A client is admitted to the hospital with symptoms consistent with right hemisphere stroke. With Neurovascular assessment requires immediate intervention by the nurse? A. Pupillary changes to ipsilateral dilation B. Orientation to person and place only. C. Left Sided Facial dropping and dysphagia D. Unequal bilateral hand grip strengths 2. Achieve maximum mobility and independence for a client multiple sclerosis (MS). Which intervention is most important for the nurse to implement? A. Provide a walker for ambulation B. Frequently assist client to the bathroom C. Apply alternating patches over the eyes D. Teach strengthing exercises 3. The Nurse is teaching a client with glomerulonephritis about self-care. Which dietary recommendations should the nurse recommend the client to follow? A. Limit oral Fluid intake to 500 mL per day B. Restrict protein intake by including meats and other high protein foods C. Increase intake of potassium-rich foods such as bananas or cantaloupe D. Increase intake of high fiber foods, such as bran cereal. 4. The nurse Is caring for a client with herpes zoster who reports painful blisters that align from the back along the chest curvature to the anterior chest. Which intervention is the highest priority for the nurse? A. Place the client on contact precaution B. Administer antiviral medication C. Place wet compresses to ruptured vesicles D. Administer narcotic analgesics 5. A young adult who suffered a severe brain injury in an automobile collision has been mechanically ventilated for the past three days and has no spontaneous respiratory effort. After serial electroencephalograms (EEG) reveal no brain activity, the healthcare provider discusses end-of-life options with family who agree to discontinue life support. Which intervention should the nurse implement? A. Ask the family if they wish would remain at bedside during withdrawal B. Request a living will be placed in the client’s medical record C. Discuss the withdrawal procedure with the family and offer support D. Turn off the mechanical ventilator and note the time of death
Med Surg Exam 2023 2 6. Following a transurethral resection of the prostate (TURP), a client is discharged from the hospital with an indwelling urinary catheter. Which instruction is most important for the nurse to include in the discharge teaching plan? A. Eliminate all the spicy food from your diet B. Drinl 3 liters of water each day C. Clamp the catheter when taking a shower D. Avoid driving a car for 2 weeks 7. On the first postoperative day, the nurse finds an older male client disoriented and trying to climb over the bed railing. Previously he was oriented to person, place, and time on admission. Which intervention should the nurse implement first? A. Apply wrist restraints B. Determine the clients blood pressure C. Administer a mild sedative D. Asses the client for pain 8. Acute soft-tissue injuries (I.e. sprains, strains) provide the nurse with a variety of teaching opportunities. Which instruction should the nurse provide to a client with a soft-tissue injury? A. Watch for shortness of breath which may indicate a fat embolus B. Begin range of motion exercises within the first 24 hours C. Apply Ice intermittently for the first 24 hours D. After edema subsides, apply heat continuously 9. A male client is admitted to the rehabilitation unit following a cerebrovascular (CVA), which resulted in paralysis of his right arm. When the nurse enters the room, he is struggling to put on a shirt, and he curses at the nurse. What is the best response from the nurse? A. “We will give you a class on dressing tomorrow” B. This unit has a policy against staff harassment C. Dressing must be a frustrating experience for you” D. “It is important to dress the right arm first” 10. A client returns to unit following a craniotomy for removal of brain tumor and is obtunded but arouses to painful stimuli. Which assessment is most important for the nurse to obtain? A. Drainage on dressing B. Last administration of analgesia C. Body temperature D. Serial blood pressure and pulse 11. An older client who is agitated, dyspneic, orthopneic, and using accessory muscle to breathe is admitted for further treatment. Initial assessment indicates beats/minute and irregular, respirations 36
Med Surg Exam 2023 3 breaths/minute, blood pressure 168/100 mmHg. Wheezes and crackles in all lung fields. An hour after the administered mg IV, which assessment should the nurse obtain to determine the client’s response to treatment? (Select all that apply) A. Skin B. Pain scale C. Lung Sounds D. Urinary output E. Oxygen saturation 12. The nurse is caring for an older male client with impaired skin integrity to sheering forces and pressure that is manifested as a draining stage 3 sacral ulcer. Which intervention is most important for the nurse to implement? A. Teach the family how to perform wound care B. Encourage a diet high in protein C. Ensure that IV fluids are administered as prescribed D. Daily Range of motion exercise 13. While planning care for a client with carpal tunnel syndrome. The nurse identifies a collaborative problem of pain. What is the etiology of this problem? A. Compression of a nerve B. Diminished blood flow C. Ischemic tissue changes D. Irritation of nerve endings 14. A young adult female visits the clinic for primary dysmenorrhea and tells the nurse that she started taking a calcium supplement to reduce her menstrual cramps. But I quit taking calcium because it caused constipation. The client to know what she does to relive her menstrual cramps. Which action should the nurse implement first to address the client’s concern? A. Encourage client to increase her dietary intake fiber B. Question the client about her use of birth control pills C. Ask her how much calcium she had been taking daily D. Determine if she takes any over-the-counter analgesics 15. A client with a medical diagnosis of a ruptured cerebral aneurysm exhibits these symptoms no eye opening, no sound vocalized, and flexion to pain (decorticate posturing). When calculating the Glasgow Coma Scale score, Which value should the nurse document for this client? A. 13 B. 9
Med Surg Exam 2023 4 C. 3 D. 5 16. A client with acute myelogenic leukemia (AML) is admitted to chemotherapy (CT) using cytarabine and the antitumor daunorubion . Which measures are most important for the nurse to implement during the induction stage of chemotherapy? A. Assessment for graft versus host disease B. Precautions to prevent infection and bleeding C. Administration of whole blood product D. Scheduling of outpatient maintenance therapy 17. To reduce pulmonary complications for a client with Amyotrophic Lateral sclerosis (ALS). Which intervention should the nurse implement? (Select all that apply) A. Perform chest physiotherapy B. Establish a regular bladder routine C. Initiate passives engage of motion exercises D. Encourage use of incentive spirometer E. Teach the client breathing exercises 18. A client with polycystic Kidney is admitted because of an abrupt onset of massive polyuria. The client is pale, tachycardia and female. Which serum laboratory finding requires immediate intervention by the nurse? A. Sodium 184 mEq/L B. Glucose 110 mg/dL C. Calcium 9 mg/dL D. HCO3 25 mEq/L 19. A client tells the nurse, “I just received good news about my tumor, I have a neoplasm, but it is benign.”. How should the nurse respond? A. Inform the healthcare provider that the client does not understand the test results B. Ask the client if the diagnostic test indicates any secondary metastasis C. Reinforce the clients joy and clarify the typical use of the team “neoplasm” D. Explain to the client the seriousness of having neoplastic disease. 20. The Nurse is assessing a client diagnosed with medical diagnosis of a Bartholin cyst. Which physical assessment technique should the nurse use to observe the cyst? A. Listen for bowel sounds in all four quarters of the abdomen B. Place the client in lithotomy position to perform a pelvic examination C. Ask the client to lie flat and cough while the nurse visualizes the inguinal area D. Expose the lesson to a woods lamp and observe for fluorescence
Med Surg Exam 2023 5 21. The nurse is preparing to administer enoxaparin 90 mg subcutaneously daily to a client with pulmonary embolism. The pharmacy provides a prefilled syringe labeled, “Enoxaparin 100 mg/1ML “How many milliliters should the nurse administer? 22. While performing a neurovascular assessment distal to a client’s fracture site, the nurse determines that the client’s pulse is present, regular, and full. Which nursing action should be taken next? A. Notify the healthcare provider of assessment finding B. Document the neurovascular assessment as normal C. Discontinue elevating the client’s affected extremity. D. Asses for color, feeling, discomfort, and movement 23. Magnesium hydroxide, 1.5 ounces P.O is prescribed for a client complaining of heartburn. After taking the prescribed dose 3 times today, how many mL of magnesium hydroxide has the client ingested? 24. A client with renal calculus is complaining of severe right flank pain, nausea, and vomiting. Which nursing problem has the highest priority? A. Acute pain related to renal calculus B. Impaired renal function to pain C. Nutritional deficit related to nausea D. Risk for aspiration related to vomiting 25. Which Technique should the nurse use when assessing for early signs of rheumatoid arthritis? A. Palpate large joints for nodules B. Palpate the lymph nodes C. Observe the skin for lesions D. Observe the clients’ fingers 26. A client with cancer develops tumor lysis syndrome (TLS) following chemotherapy. Wich nursing action has the highest priority in responding to the symptoms of this syndrome? A. Identify potential sources of infection B. Maintain intravenous therapy C. Instruct the client to take analgesics on a regular schedule. D. Encourage the client to verbalize anxiety and grief
Med Surg Exam 2023 6 27. A client with Hepatitis A is complaining of weakness and chronic fatigue. Which intervention is most important for the nurse to implement? A. Provide liberal fluids for hydration and excretion of metabolic waste products B. Place belongings with client reach so bed rest can be maintained C. Encourage dietary selections that are high in essential vitamins and Iron D. Ensure the client has scheduled rest periods every 4 to 6 hours during the day 28. A client who had a cast yesterday to the lower left arm comes to the clinic complaining of pain in the cast arm. Which assessment finding is most important for the nurse to identify? A. Presence of a pressure ulcer under the cast B. Location of burning pain below the cast C. Circulatory impairment distal to the cast D. 29. The nurse review lab values of a female client with metastatic breast cancer and notes that the client’s serum calcium level is 14 mg/dL. The client is weak, fatigued, and depressed. New prescriptions include increasing the rate of intravenous fluids. Which action should the nurse take first? A. Increase the intravenous fluids as prescribed. B. Offer to provide privacy so the client can rest C. Encourage verbalization of the clients’ feelings D. Provide a nutritional supplement for a snack 30. A client with hypovolemic shock is admitted to the intensive care unit with an intraosseous (IO) vascular access device placed in the right proximal tibia. The client has received two liters of normal saline and one unit of packed red blood cells through the IO access device since admission. Which assessment finding warrants immediate intervention by the nurse? A. Client reports tenderness at intraosseous insertion site B. Client verbalizes feeling tightness in right calf muscle. C. IO Vascular access in lace greater than 24 hours D. Sluggish intraosseous blood return when aspirated 31. A client has a neutrophil count of (ANC) of 500/mm3 (0.5 x 109/L) after completing chemotherapy. Which intervention is most important for the nurse to implement? A. Implement bleeding precaution B. Review needs for pneumococcal vaccine C. Asses Vital signs every 4 hours D.
Recommended for you

Document continues below
20
ATI med surg assessment-with 100% verified solutions-2023-2024
ATI med surg assessment-with 100% verified solutions-2023-2024
swift river
100% (9)
5
NRNP 6665 Midterm Exam 2023, Walden University, NRNP6665 Midterm Exam, NRNP 6665 Week 6 Midterm Exam, NRNP6665 Week 6 Midterm Exam
NRNP 6665 Midterm Exam 2023, Walden University, NRNP6665 Midterm Exam, NRNP 6665 Week 6 Midterm Exam, NRNP6665 Week 6 Midterm Exam
Medical
50% (6)
4
NRNP 6645 Final Exam 2023, Walden University, NRNP6645 Final Exam, NRNP 6645 Week 11 Final Exam, NRNP6645 Week 11 Final Exam
NRNP 6645 Final Exam 2023, Walden University, NRNP6645 Final Exam, NRNP 6645 Week 11 Final Exam, NRNP6645 Week 11 Final Exam
Medical
None
6
NRNP 6560 Final Exam 2023 (3 Versions), Walden University, NRNP6560 Final Exam, NRNP 6560 Week 11 Final Exam, NRNP6560 Week 11 Final Exam
NRNP 6560 Final Exam 2023 (3 Versions), Walden University, NRNP6560 Final Exam, NRNP 6560 Week 11 Final Exam, NRNP6560 Week 11 Final Exam
Medical
None
Med Surg Exam 2023 7 32. Clients’ laboratory findings indicate elevations in thyroxine and triiodothyronine hormones. The nurse suspects that the client may have hyperthyroidism. Which symptom is most often associated with hyperthyroidism? A. Atrophied thyroid gland B. Increased pulse rate C. Periorbital edema D. Diarrhea Stools 33. Prior to initiating peritoneal dialysis, which nursing action is most important for the nurse to implement? A. Determine the client’s oxygen saturation B. Obtain and record the clients’ vital signs C. Ascultate the clients’ vital signs D. Observe the amount and color of the client’s urine 34. The nurse is preparing to insert an indwelling catheter for a male client who has diabetes and a semirigid penile implant. After placing the sterile drapes and prepping the meatus, the nurse notes that the client’s penis is erect. Which action should the nurse implement? A. Ask the client to deflate the implant B. Talk to the client about his implant C. Continue to insert the catheter D. Wait until the erection subsides 35. Which finding should the nurse document as primary manifestation of osteoporosis in an older woman? A. Loss of height over time B. Decreased serum calcium level C. Pain in the spine and neck D. Abnormal cardiac status in the ECG 36. The nurse is conducting discharge teaching for a male client with a prescription for magnesium hydroxide 15 mL one time per day. His home medication cup is ounces. How many ounces should he take each dose? A. 0.5 ounces B. 0.05 ounces C. 0.25 ounces D. 1 ounce 37. Which change in lab values would indicate to the nurse that treatment for gout is successful?
Med Surg Exam 2023 8 A. Increased Serum uric acid B. Decreased serum purine C. Increased serum purine D. Decreased serum uric acid 38. A client with acute renal injury (AKI) weighs 50 kg and has potassium level of 6.7 mEq/L (6.7 mmol/L) is admitted to the hospital. Which prescribed medication should the nurse first administer? A. Sevelamer one tablet PO B. Calcium acetate one tablet PO C. Sodium polystyrene sulfonate 15 grams PO D. Epotin alfa, recombinant 2,500 Units SUBQ 39. Following the administration of intravenous regular insulin to a client diagnosed with hyperkalemia, the nurse should expect which outcome to occur? A. A temporary shift of potassium into the cells B. An increase of potassium in the urinary output C. An improvement in the cardiac conduction abnormalities D. Excretion of potassium via bowel movement 40. An adult male who is insulin dependent diabetic. Is admitted to the hospital because of headaches. When the client stiffens and begins to seize. Which intervention is most important for the nurse to implement? A. Pad all side rails with available pillows and blankets B. Determine the clients blood glucose level C. Give the client a rapid form of glucose supplement D. Obtain a suction set-up in the room 41. A young adult is burned when wearing a shirt that was splashed with lighter fluid and caught. The shirt immediately without unbuttoning. What should the nurse implement first? A. Assess range of motion B. Monitor pulse intensity C. Evaluate extremity sensation D. 42. A female client returns to the clinic after being treated for chlamydia with azithromycin IM and reports that she still has symptoms. The healthcare provider obtains a swab of the discharge from the cervix for testing chlamydia. The client reports maintaining a monogamous relationship when laboratory results
Med Surg Exam 2023 9 are positive for sexually transmitted infection. Which information should the nurse obtain to evaluate the ineffective results of treatment? A. Determine if the clients sexual partner received treatment for chlamydia B. Ask the client if the complete course of antibiotics was completed C. Confer with the healthcare provider about a different course of antibiotics D. Inquire further about all sexual encounters and any other sexual activity 43. The nurse implements a change in the approach to the client care after gathering evidence of a new approach. What should the nurse do first? A. Engage staff in evidence-based practice B. Consult with clinical nursing expert C. Revise clinical practice guidelines D. Evaluate effectiveness of the change 44. The Home Health nurse is caring for a client with Parkinson’s disease who is beginning to experience swallowing difficulties. Which intervention should the nurse include for this client? A. Teach the client to take his medication an hour before meals to enhance the swallowing reflex B. Tell the client to lay on his left side to prevent his tongue from falling back in his mouth C. Prepare the client and the family for the future need of a gastrostomy tube for feeding D. Encourage the client and family to provide a semi-solid diet with thick liquids 45. A client with chronic cirrhosis has esophageal varices. it is most important for the nurse to monitor the client for? A. Hematemesis B. Brown foamy urine C. Anorexia D. Clay colored stool 46. The intracranial pressure of a brain-injured client who is on a ventilator has increased from 15 mm Hg to 25 mm Hg within the last 30 minutes. The client is beginning to flex all extremities intermittently. Based on these findings, which immediate action should the nurse take? A. Draw stat arterial gases to the hypercapnia B. Manually ventilate the client using and ambu bag C. Asses the patency of the client’s artificial airway D. 47. A young male client has a diagnosis of epididymitis and a positive culture for Escherichia coli. Which information should the nurse include in the teaching plan? A. Epididymitis is a pre-cancerous condition B. Avoid penile contact with rectal area
Med Surg Exam 2023 10 C. Surgical intervention often indicated D. Obtain an annual prostate digital exam 48. The ESR (sedimentation rate) of a client being treated with corticosteroids for rheumatoid arthritis has decreased. Which explanation should the nurse provide the client to explain this change in lab values? A. The treatment so far has not been effective B. A value of 0 will indicate that that the client is cured C. The client is most likely responding to treatment D. The client disease is currently in a remission 49. A client has a prescription for a viscous compound containing lidocaine HCL and diphenhydramine to relieve the discomfort of mucositis caused by radiation therapy. Which instructions should the nurse provide the client about administration of this prescription? A. Saturate a sterile dressing with the solution and pack the wound lightly B. Dab the solution over the reddened areas and cover the site with occlusive dressing C. Gently pat the solution on the sore areas, using cotton tipped applicators D. Swish the solution around in the mouth, and swallow the remaining solution 50. A client admitted dehydration resulting from vomiting and diarrhea. The nurse knows that the client is at greatest risk of developing which condition? A. Bowel perforation B. Papilledema C. Tinnitus D. Cardiac dysrhythmia 51. When the nurse begins discharge instructions for a client and her spouse, the client who had an above the knee amputation for complications associated with diabetes, tells the nurse that she is not ready to go home and wants to stay home in the hospital another day, which intervention is important for the nurse to implement? A. Explain the take home medications that can help the client manage her anxiety B. Tell the spouse to wait outside the room so the nurse can interview the client alone C. Ask the client what frightens her about leaving the hospital and returning home D. Review the details of the home health care plan devised by the multidisciplinary team 52. The nurse admits a client who has a medical diagnosis of bacterial meningitis to the unit. Which intervention has the highest priority when providing care for the client?
Med Surg Exam 2023 11 A. Obtain results of culture and sensitivity of CSF B. Administer initial dose of broad-spectrum antibiotic C. Instruct the client to force fluids hourly D. Asses the client for symptoms of hyponatremia 53. While performing assisted range of motion exercises for a client with osteoarthritis, the nurse notes joint crepitus. Which action should the nurse take? A. Notify the healthcare provider of findings B. Continue the range of motion exercises C. Immobilize the extremity joint D. Apply moist heat to the site 54. A female client with metastatic breast cancer is admitted with shortness of breath and pleural effusion. The client has a living will and the family is requesting hospice information. Which information should the nurse provide regarding hospice? (Select all that Apply) A. Can be provided within comfort of home B. Provides comfort, dignity, and emotional support C. Hospice services can be initiated prior to discharge D. A living will become invalid when receiving hospice care E. Family members can be involved in the plan of care

MED-SURG HESI practice questions from the Saunders bookThe nurse is preparing to assist a client with a cuffed tracheostomy tube to eat. What intervention is the priority before the client is permitted to drink or eat? Inflate the cuffRationale: If a client with a tracheostomy is allowed to eat and the tracheostomy has a cuff, the nurse should inflate the cuff to prevent aspiration of food or fluids. The cuff would not be deflated because of the risk of aspiration.The nurse has implemented a bowel maintenance program for an unconscious client. The nurse would evaluate the plan as best meeting the needs of the client if which method was successful in stimulating a bowel movement? Glycerin suppositoryRationale: The least amount of invasiveness needed to produce a bowel movement is best. Use of glycerin suppositories is the least invasive method and usually stimulates bowel evacuation within a half-hour.A client is readmitted to the hospital with dehydration after surgery for creation ofan ileostomy. The nurse assesses that the client has lost 3 lb of weight, has poor skin turgor, and has concentrated urine. The nurse interprets the client’s clinical picture as correlating most closely with recent intake of which medication, which is contraindicated for the ileostomy client? BiscodylRationale: The client with an ileostomy is prone to dehydration because of the location of the ostomy in the gastrointestinal tract and should not take laxatives.The client is complaining of skin irritation from the edges of a cast applied the previous day. Which action should the nurse take? The nurse petals the edges of the cast with tape Rationale: minimize skin irritation. The nurse is taking a health history for a client with hyperparathyroidism. Which question would elicit information about this client’s condition? “Are you experiencing pain in your joints?”Rationale: Hyperparathyroidism is associated with over secretion of parathyroid hormone (PTH), which causes excessive osteoblast growth and activity within the bones. When bone reabsorption is increased, calcium is released from the bones into the blood, causing hypercalcemia. The bones suffer demineralization as a result of calcium loss, leading to bone and joint pain
A client with type 2 diabetes mellitus has a blood glucose level greater than 600 mg/dL (34.3 mmol/L) and is complaining of polydipsia, polyuria, weight loss, and weakness. The nurse reviews the health care provider’s documentation and expects to note which diagnosis? Hyperosmolar hyperglycemic syndrome (HHS)Rationale: HHS is seen primarily in clients with type 2 diabetes mellitus, who experiencea relative deficiency of insulin. The onset of signs and symptoms may be gradual. Manifestations may include polyuria, polydipsia, dehydration, mental status alterations, weight loss, and weakness.The nurse is developing a plan of care for a client who will be admitted to the hospital with a diagnosis of deep vein thrombosis (DVT) of the right leg. The nurse develops the plan, expecting that the health care provider (HCP) will most likely prescribe which option? —Maintain activity level as prescribed.Rationale: Standard management for the client with DVT includes maintaining the activity level as prescribed by the health care provider; limb elevation; relief of discomfort with warm, moist heat; and analgesics as needed. Recent research is showing that ambulation, as previously thought, does not cause pulmonary embolism and does not cause the existing DVT to worsen. Therefore, the nurse should maintain the prescribed activity level, which could be bed rest or ambulation. Opioid analgesics are not required to relieve pain, and pain normally is relieved withacetaminophen.re provider (HCP) will most likely prescribe which option? Maintain activity level as prescribed.A client has had surgery to repair a fractured left hip. When repositioning the client from side to side in the bed, what should the nurse plan to use as the most important item for this maneuver? —– Abductor splintRationale: After surgery to repair a fractured hip, an abductor splint is used to maintain the affected extremity in good alignment. A bed pillow and an overhead trapeze also areused, but neither is the priority item to be used in repositioning the client from side to side.The nurse is preparing to care for a client who had a supratentorial craniotomy. The nurse should plan to place the client in which position ? Semi Fowler’sRationale: you know why
A client with an external arteriovenous shunt in place for hemodialysis is at risk for bleeding. Which is the priority nursing intervention? — Ensure that small clamps are attached to the arteriovenous shunt dressing.Rationale: An external arteriovenous shunt is a less common form of access site but carries a risk for bleeding when it is used because 2 ends of an external cannula are tunneled subcutaneously into an artery and a vein, and the ends of the cannula are joined. If accidental disconnection occurs, the client could lose blood rapidly. For this reason, small clamps are attached to the dressing that covers the insertion site for use ifneeded. The shunt site also should be assessed at least every 4 hours. Checking the shunt for the presence of bruit and thrill relates to patency of the shunt. Although checking the results of the prothrombin time is important, it is not the priority nursing action.A client is admitted to the hospital emergency department after receiving a burn injury in a house fire. The skin on the client’s trunk is tan, dry, and hard. It is edematous but not very painful. The nurse determines that this client’s burn should be classified as which type? Full-thicknessRationale: Full-thickness burns involve the epidermis, the full dermis, and some of the subcutaneous fat layer. The burn appears to be a tan or fawn color, with skin that is hard, dry, and inelastic. Edema is severe, and the accumulated fluid compresses tissue underneath because of eschar formation. Some nerve endings have been damaged, and the area may be insensitive to touch, with little or no pain.The nurse is assessing a client with a duodenal ulcer. The nurse interprets that which sign or symptom is most consistent with the typical presentation of duodenal ulcer? Pain that is relieved by food intakeRationale: The most typical finding with duodenal ulcer is pain that is relieved by food intake. The pain is often described as a burning, heavy, sharp, or “hunger pang” pain that often localizes in the mid-epigastric area.
The nurse is developing a plan of care for a client with Cushing’s syndrome. The nurse documents a client problem of excess fluid volume. Which nursing actions should be included in the care plan for this client? Select all that apply. Monitor daily weight. Monitor intake and output. Assess extremities for edemaRationale: The client with Cushing’s syndrome and a problem of excess fluid volume should be on daily weights and intake and output and have extremities assessed for edema. He or she should be maintained on a high-potassium, low-sodium diet.A client with a history of ear problems is going on vacation by aircraft. The nurse advises the client to include which activities to prevent barotrauma during ascentand descent of the airplane? Select all that apply: Yawning- Swallowing – Chewing gum – Sucking on a hard c ( its not what your thinking )Rationale:Clients who are prone to barotrauma should perform any of a variety of mouth movements to equalize pressure between the ear and the atmosphere, particularly during ascent and descent of an aircraft. These can include yawning, swallowing, drinking, chewing, and sucking on hard candy. Valsalva maneuver also may be helpful. The client should avoid sitting with the mouth motionless during this time because the resulting lack of pressure change in the ear will contribute to pressure buildup behind the tympanic membrane.The nurse is assessing a client with an abdominal aortic aneurysm. Which assessment finding by the nurse is unrelated to the aneurysm? BA. Pulsatile abdominal massB. Hyperactive bowel sounds in the areaC. Systolic bruit over the area of the massD. Subjective sensation of “heart beating” in the abdomen
Rationale: Hyperactive bowel sounds are not related specifically to an abdominal aortic aneurysm. Not all clients with abdominal aortic aneurysm exhibit symptoms. Those who do may describe a feeling of the “heart beating” in the abdomen when supine or being able to feel the mass throbbing. A pulsatile mass may be palpated in the middle and upper abdomen. A systolic bruit may be auscultated over the mass.The nurse is evaluating the condition of a client after pericardiocentesis performed to treat cardiac tamponade. Which observation would indicate that the procedure was effective? A rise in blood pressureFollowing pericardiocentesis, the client usually expresses immediate relief. Heart sounds are no longer muffled or distant and blood pressure increases.The nurse is creating a plan of care for a client with a diagnosis of nephrotic syndrome whose glomerular filtration rate (GFR) is normal. Which interventions should the nurse include in the plan of care? Select all that apply. All of theseMonitor daily weight.Maintain sodium restrictions.Monitor intake and output (I&O).Maintain bed rest when edema is severe.Rationale:Controlling edema is a critical aspect of therapeutic management of nephrotic syndrome. If the GFR is normal, dietary intake of proteins is needed to restore normal plasma oncotic pressure and thereby decrease edema. Daily measurement of weight and abdominal girth, and careful monitoring of I&O will determine whether weight loss is caused by diuresis or protein loss. Dietary modifications may include salt restriction and fluid restriction and are based on the client’s symptoms. Bed rest is prescribed to promote diuresis when edema is severe.The nurse is monitoring the chest tube drainage system in a client with a chest tube. The nurse notes intermittent bubbling in the water seal compartment. Whichis the most appropriate action? Document the findingsRationale: Bubbling in the water seal compartment is caused by air passing out of the pleural space into the fluid in the chamber. Intermittent bubbling is normal. It indicates that the system is accomplishing one of its purposes, removing air from the pleural space. Therefore, it is unnecessary to call the HCP or change the chest tube drainage system. Continuous bubbling during inspiration and expiration indicates an air leak. If this occurs, it must be corrected.
The nurse is preparing for removal of an endotracheal (ET) tube from a client. In assisting the health care provider with this procedure, which is the initial nursing action? She had betta: Suction the ET tube.Rationale: Once the client has been weaned successfully and has achieved an acceptable level of consciousness to sustain spontaneous respiration, an ET tube may be removed. The ET tube is suctionedfirst, and then the cuff is deflated and the tube is removed.The nurse determines that a client with a tracheostomy tube needs suctioning if which finding is noted? Rhonchi are auscultated.Rationale: When you auscultate that rhonchi, it is an indication that there are secretions in the large airways. The client requires suctioning if the client cannot expectorate them. A pulse oximetry reading of 96% is an acceptable reading. A pleural friction rub is indicative of inflamed pleural surfaces. Fine crackles are indicative of air moving into previously deflated alveoli.The nursing student is assigned to care for a client with a diagnosis of acute kidney injury (AKI), diuretic phase. The nursing instructor asks the student about the primary goal of the treatment plan for this client. Which goal, if stated by the nursing student, indicates an adequate understanding of the treatment plan for this client ? Prevent loss of electrolytes.Rationale: In the diuretic phase , fluids and electrolytes are lost in the urine . As a result,the plan of care focuses on fluid and electrolyte replacement and monitoring.The nurse has been caring for a client who required a Sengstaken-Blakemore tube because other treatment measures for esophageal varices were unsuccessful. The health care provider (HCP) arrives on the nursing unit and deflates the esophageal balloon. Which assessment finding by the nurse is the most important and should be reported to the HCP immediately? HematemesisRationale: A Sengstaken-Blakemore tube may be inserted in a client with a diagnosis of cirrhosis with bleeding esophageal varices. It has both an esophageal and a gastric balloon. The esophageal balloon exerts pressure on the ruptured esophageal varices and stops the bleeding. The pressure of the esophageal balloon is released at intervals to decrease the risk of trauma to esophageal tissues, including esophageal rupture or necrosis. When the balloon is deflated, the client may begin to bleed again from the esophageal varices, manifested as vomiting of blood (hematemesis).
The nurse is planning discharge teaching for a client diagnosed and treated for compartment syndrome. Which information should the nurse include in the teaching? “Bleeding and swelling caused increased pressure in an area that couldn’t expand.”Rationale: Compartment syndrome is caused by bleeding and swelling within a tissue compartment thatis lined by fascia, which does not expand. The bleeding and swelling put pressure on the nerves, muscles,and blood vessels in the compartment, triggering the symptoms.The nurse has just admitted to the nursing unit a client with a basilar skull fracture who is at risk for increased intracranial pressure. Pending specific healthcare provider prescriptions, the nurse should safely place the client in which positions? Select all that apply.Head midlineNeck in neutral positionHead of bed elevated 30 to 45 degreesRationale:Use of proper positions promotes venous drainage from the cranium to keep intracranial pressure from elevating. The head of the client at risk for or with increased intracranial pressure should be positioned so that it is in a neutral, midline position. The head of the bed should be raised to 30 to 45 degrees. The nurse should avoid flexing or extending the client’s neck or turning the client’s head from side to side.The nurse is caring for a postoperative pneumonectomy client. Which finding on assessment of the client is an adverse sign or symptom indicating pulmonary edema? Lung crackles in the remaining lungRationale: The client with pulmonary edema that developed after pneumonectomy demonstrates dyspnea, cough, frothy sputum, crackles, and possibly cyanosis.A client is scheduled for surgical creation of an internal arteriovenous (AV) fistulaon the following day. The client says to the nurse, “I’ll be so happy when the fistula is made tomorrow. This means I can have that other hemodialysis catheter pulled right out.” Which interpretation should the nurse make based on the client’s statement?The client does not understand that the site needs to mature or develop for 1 to 2 weeks before use.
Rationale: An AV fistula is the internal creation of an arterial-to-venous anastomosis. This causes engorgement of the vein, allowing both the artery and the vein to be easily cannulated for hemodialysis. Fistulas take 1 to 2 weeks to mature (engorgement) or develop before they can be used for dialysis, so the current method of access must remain in place to be used during that period.A client is admitted with suspected diabetic ketoacidosis (DKA). Which clinical manifestations best support a diagnosis of DKA? 1.Blood glucose 500 mg/dL (27.8 mmol/L); arterial blood gases: pH 7.30, PaCo2 50, HCO3– 26.2.Blood glucose 400 mg/dL (22.2 mmol/L); arterial blood gases: pH 7.38, PaCo2 40, HCO3– 22.3.Blood glucose 450 mg/dL (25.0 mmol/L); arterial blood gases: pH 7.48, PaCo2 39, HCO3– 29.4.Blood glucose 350 mg/dL (19.4 mmol/L); arterial blood gases: pH 7.28, PaCo2 30, HCO3– 14.Rationale: DKA is caused by a profound deficiency of insulin and is characterized by hyperglycemia (blood glucose level greater than or equal to 250 mg/dL [13.9 mmol/L]), ketosis (ketones in urine or serum), metabolic acidosis, and dehydration. The correct option is 4, as it represents an elevated blood glucose and the arterial blood gases (ABGs) indicate metabolic acidosis. Option 1 is incorrect, as the ABGs indicate respiratory acidosis; option 2 is incorrect, as the ABG values are within normal; and option 3 is incorrect, as the ABGs indicate metabolic alkalosis.A client with ulcerative colitis has a prescription to begin a salicylate compound medication to reduce inflammation. What instruction should the nurse give the client regarding when to take this medication? After mealsRationale: The medication needs to be taken after meals to reduce gastrointestinal irritationA client with chronic kidney disease who is scheduled for hemodialysis this morning is due to receive a daily dose of enalapril. When should the nurse plan toadminister this medication? On return from dialysisRationale: Antihypertensive medications such as enalapril are given to the client following hemodialysis. This prevents the client from becoming hypotensive during dialysis and from having the medication removed from the bloodstream by dialysis.The nurse is monitoring the function of a client’s chest tube that is attached to a drainage system. The nurse notes that the fluid in the water seal chamber rises
with inspiration and falls with expiration. The nurse determines that which is occurring? Tidaling is present. ( it supposed to happen)Rationale: When the chest tube is patent, the fluid in the water seal chamber rises with inspiration and falls with expiration. This is referred to as tidaling and indicates proper function of the system.A client with acute ulcerative colitis requests a snack. Which is the most appropriate snack for this client?1.Carrots and ranch dip2.Whole-grain cereal and milk3.A cup of popcorn and a cola drink4.Applesauce and a graham crackerRationale: The diet for the client with ulcerative colitis should be low fiber (low residue). The nurse shouldavoid providing foods such as whole-wheat grains, nuts, and fresh fruits or vegetables. Typically, lactose-containing foods also are poorly tolerated. The client also should avoid caffeine, pepper, and alcohol.The nurse is planning to teach a client with peripheral arterial disease about measures to limit disease progression. Which items should the nurse include on a list of suggestions for the client? Select all that apply. (2,4,5)1.Soak the feet in hot water daily.2.Be careful not to injure the legs or feet.3.Use a heating pad on the legs to aid vasodilation.4.Walk each day to increase circulation to the legs.5.Cut down on the amount of fats consumed in the diet.Rationale:Long-term management of peripheral arterial disease consists of measures that increase peripheral circulation (exercise), promote vasodilation (warmth), relieve pain, and maintain tissue integrity (foot care and nutrition). Soaking the feet in hot water and application of a heating pad to the extremity are contraindicated. The affected extremity may have decreased sensitivity and is at risk for burns. Also, the affected tissue does not obtain adequate circulation at rest. Direct application of heat raises oxygen and nutritional requirements of the tissue even further.
A client who has undergone gastric surgery has a nasogastric (NG) tube connected to low intermittent suction that is not draining properly. Which action should the nurse take initially? (3)1.call the surgeon to report the problem.2.Reposition the NG tube to the proper location.3.Check the suction device to make sure it is working.4.Irrigate the NG tube with saline to remove the obstruction.Rationale:After gastric surgery, the client will have an NG tube in place until bowel function returns. It is important for the NG tube to drain properly to prevent abdominal distention and vomiting. The nurse must ensure that the NG tube is attached to suction at the level prescribed and that the suction device is working correctly. The tip of the NG tube may be placed near the suture line. Because of this possibility, the nurse should never reposition the NG tube or irrigate it. If the NG tube needs to be repositioned, the nurse should call the surgeon, who would do this repositioning under fluoroscopyThe community health nurse is visiting a homeless shelter and is assessing the clients in the shelter for the presence of scabies. Which assessment finding should the nurse expect to note if scabies is present? Brown-red macules with scalesPustules on the trunk of the bodyWhite patches noted on the elbows and kneesMultiple straight or wavy threadlike lines underneath the skinRationale:Scabies can be identified by the multiple straight or wavy threadlike lines beneath the skin. The skin lesions are caused by the female, which burrows beneath the skin to lay its eggs. The eggs hatch in a fewdays, and the baby mites find their way to the skin surface, where they mate and complete the life cycle. Options 1, 2, and 3 are not characteristics of scabies.
The registered nurse is precepting a new nurse who is caring for a client with pernicious anemia as a result of gastrectomy. Which statement made by the new nurse indicates understanding of this diagnosis? “Decreased production of intrinsic factor by the stomach affects absorption of vitamin B12 in the small intestine.” Rationale:Intrinsic factor is produced in the stomach but is used to aid in the absorption of vitamin B12 in the small intestine.The nurse has a prescription to obtain a urinalysis specimen from a client with anindwelling urinary catheter. Which actions should the nurse include in performingthis procedure? Select all that apply. (1,2,3,5)1.Explaining the procedure to the client2.Clamping the tubing of the drainage bag3.Aspirating a sample from the port on the drainage tubing4.Obtaining the specimen from the urinary drainage bag5.Wiping the port with an alcohol swab before inserting the syringeRationale:A urine specimen is not taken from the urinary drainage bag. Urine undergoes chemical changes while sitting in the bag, so its properties do not necessarily reflect current client status. In addition, it may become contaminated with bacteria from opening the system. The remaining options are correct interventions for obtaining the specimen.

Leave a Comment

Scroll to Top